Этого треда уже нет.
Это копия, сохраненная 5 мая 2019 года.

Скачать тред: только с превью, с превью и прикрепленными файлами.
Второй вариант может долго скачиваться. Файлы будут только в живых или недавно утонувших тредах. Подробнее

Если вам полезен архив М.Двача, пожертвуйте на оплату сервера.
69.jpg54 Кб, 1280x720
Тред вопросов и ответов 456196 В конец треда | Веб
Перед тем как задать тупой вопрос и получить не менее тупой ответ - подумой.

Предыдущий >>454356 (OP)

Нормальный иностранный научпоп, где есть понятные ответы на большинство популярных вопросов на тему физики:
https://www.youtube.com/watch?v=Y7Ac8zKTD-E&t=426s
https://www.youtube.com/watch?v=gvX29HPmBEI
https://www.youtube.com/watch?v=sB1EPGmpzyg
https://www.youtube.com/user/1veritasium/featured

На русском лучше искать лекции различных вузовских преподов и т.п., Топлесы и прочее идут строго мимо.
2 456206
Жобрый вечер, двачик.

Хочу научиться проектировать реактивные двигатели самостоятельно. Звучит смешно, но я закончил универ по похожей специальности, могу делать статические и динамические расчёты на прочность, расчёты газов и тд, и всё все то же на соответствующем софте, так что считаю это возможным.

Так вот, возможно в треде совершенно случайно окажется человек, который знаком с тематикой и может мне посоветовать литературу? Нагуглить что-то годное по теме нереально, даже простейшие концептуальные расчёты вроде какой тяги достаточно для подъёма такото-то планера самолёта, или количество воздуха продуваемое через двигатель при такой-то тяге, и тп. Ну вы поняли, был бы очень благодарен.
3 456219
4 456220
>>56206
В spc вроде есть тред
5 456234
Допустим, что существуют инопланетные цивилизации. Какова вероятность того, что у них тоже есть/были компы на бинарной логике? Насколько это очевидная технология, а развитии можно пройти мимо нее?
6 456246
>>56234
Ну давай поспекулируем. Организовать компьютер на бинарной логике проще, чем на какой-либо другой, ящитаю. В СССР был комп на тройной логике, но, судя по тому, что он не взлетел, это менее эффективно.
7 456270
>>56246
Хм, возможно, не совсем так выразился. Меня интересует скорее, насколько это common knowledge - использовать для работы компьютера кучу маленьких переключателей с позициями "0" и "1" и получать из них результат. Какова вероятность, что компы инопланетян работали/работают на куче таких переключателей, а не, например, используют для работы атомарные колебания какого-нибудь водорода, например, где совершенно иной принцип обсчета данных и получения результата.
8 456272
Гайс такой вопрос. Что есть время ? Не вижу в своей тупой бошке основания оценки времени как нечто субстанциональное. Мне представляется что время есть условная абстракция , просто слово. Я думаю что говорить так неверно, но слово обобщающее , а не определяющее. Подскажите в чем я не прав пожалуйста
9 456273
>>56272

>Что есть время ?


Cлово.
10 456280
Саентач, прошу тебя разъяснить вопрос о синхроничности Юнга. Я не хочу ударяться в мистику, поэтому пришел сюда. Сам отношусь скептически к этой хуйне, но неоднократно замечал "странные" совпадения. Особенно остро вопрос встал, когда, когда подруга моей тян, любительница погадать на картах Таро, делала прогноз моей тян на более-менее ближайшее будущее. Разумеется, я ждал разговоры о "злых языках", но она говорила довольно определенные вещи, например, сказала, что у нее будет операция, что перед этим неожиданно придут деньги и еще что-то. Так вот, время проходит, месяц, два, моя тян пошла на обследование в поликлинику. И к всеобщему удивлению у нее кое-что находят и ей действительно назначают операцию. До момента, когда она легла в больницу, она занималась мелкой торговлей по интернету. И у нее возник спор с каким-то старым пердуном-покупателем, который намекал, что она его наебать решила. В итоге он выслал ей сумму за товар, она ему отправляет заказ. И через несколько дней ей приходит такая же сумма от него без каких-либо комментариев. Ей как раз деньги были нужны там было больше 20к, она их не трогала, но он год не отзывался и она их уже заюзала. Я не спец в картах, но так ли это вероятно, что в целой колоде выпадают карты с именно таким смыслом? Или дело в подружкиных словах? Потом были еще странности, я не хочу перечислять, это самые яркие. Я с тех пор нахожусь в замешательстве.
11 456282
>>56280
Погугли когнитивные искажения, у тебя целый букет их, лол.
12 456283
>>56282
Я знаю, что это, но все же ждал объяснение
13 456287
Посмотрел тут лекцию по эволюции и задумался как так вышло что с течением времени сапиенсы увеличивали продолжительность жизни, возникло следующие предположение.

Вот есть два детеныша человека умелого один условно взрослеет в 10 лет и становится половозрелым самцом, а второй отстает в созревании и более менее сформировывается к 12.
Первого уже в в 10 лет выкидывают на мороз заставляют идти на охоту, бороться с хищниками и себе подобными за самку, в мозгу его бушуют гормоны и он обучается уже не так эффективно. Второй в это время впитывает знания об окружающем мире от бабки и наблюдая за всеми со стороны.

В итоге первый детеныш в силу малого опыта погибает не оставив потомства с большей вероятностью. Второй же имеет больше преимуществ и шанс выжить. Вооот.
Таким образом идет естественный отбор по продолжительности детства и жизни у сапиенсов.
Потом возника еще одна мысль, а что если сейчас в современном мире в связи с тем что возраст готовности индивида к самостоятельной жизни определяется паспортом, а не биологией, то данный отбор прекратился, и более того возможно направлен в противоположную сторону. Типо раньше созрел и уже в 13 лет начал плодить потомство. Отсюда такое большое количество акселератов в школах. Хотя тут скорее больший вклад делает изобилие пищи, но все же.

Современная медицина конечно не даст акселератам откинуть копыта в 50 лет, но если я все верно расписал, то естественного прироста продолжительности жизни нам не видать.
14 456297
>>56287

>и задумался как так вышло что с течением времени сапиенсы увеличивали продолжительность жизни


Никак не увеличивали. Сразу развились от долгоживущей ветви приматов, потому что отбор по интеллекту мог появиться только у долгоживущего вида.
Потом в медных веках просрали всю долгую жизнь, потому что начали экспериментировать с ядами и стрессами. Теперь научились жить так же долго, как до бронзового века. Никакого прироста не было, был бой правой руки с левой.
15 456298
>>56270
Мне кажется это очень вероятно, так как счеты у всех одинаковые получались
16 456299
>>56272

>Мне представляется что время есть условная абстракция , просто слово.


Пространство тоже условная абстракция, но ты не задумывался об этом, потому что оно нагляднее. (хотя для физики и время и пространство однохуйственно выглядят)
17 456306
>>56297
ну а как все человекообразные обезьяны стали долгоживущими? Не думаю что вские предки типо проконсулов по 50 лет жили.
18 456309
Есть ли какая-нибудь реалистичная возможность того, что в ближайшие 20-30 лет будут открытия в физике уровня открытия электричества или радиоактивности? То есть что-то, что реально изменяет цивилизацию а не просто корректировка десятого знака после запятой в какой-нибудь константе.
19 456310
>>56309
Нет. Следующее большое открытие будет уже в следующем веке. Анлаки.
20 456312
>>56309
Конечно, темное вещество но это не точно но многие верят
21 456318
>>56298
У кого у всех? Если ты говоришь о народах земли то не одинаковые
22 456319
>>56287

>возраст готовности индивида к самостоятельной жизни определяется паспортом, а не биологией


чтоблять

>тсюда такое большое количество акселератов в школах


чтоблятьx2
Какая связь между ростом и возрастом зачатия потомства?
23 456326
>>56319
Никто тебя на работу в 15 лет не возьмёт.
У меня в 2007 в 11 классе славянин с бородой уже был, у аксклератов не только рост больше.
24 456341
сап наукач, зачем нужны взаимозаменяемые конусы для колб и как они выглядят?
25 456352
>>56309
Если ускоритель на основе этой технологии сделают, может откроют гравитоны
https://www.youtube.com/watch?v=UZbXGDxMRCw
15466458065440.png581 Кб, 1080x2280
26 456353
Absorbance это где и в чем?
>>456140 чего блядь?
27 456356
>>56196 (OP)
На харкачике оказывается нет доски про экономику. Попробовал зайти в /по со своим вопросом, но там какой-то лютый пиздец. Поэтому спрошу здесь.

Что ученый анон думает об универсальном базовом доходе?
Конкретно я хотел узнать мнение лево_настроенных товарищей.
С одной стороны это же как бы распределение, равенство, возможности для образования, хорошо одним словом. Но с другой стороны это же не решает фундаментальных проблем неравенства - билли гейтс, например, так же получать будет этот базовый доход, сверху своих миллиардов. По-моему это полумера. Причем выгодная именно "магнатам", наебка работяг.
Сам я не отношу себя ни к комми, ни к капиталистам. Я просто наблюдаю со стороны.
28 456357
Я верующий гностик. Недавно во время спора в интернете я сказал, что готов отказаться от веры когда мир будет полностью описан наукой(если это будет). На что получил интересный аргумент: такого никогда уже не случится т. к. квантовую случайность можно оправдывать Божьей волей или судьбой, т. е. строить религию исходя из этого момента. Скажите, пожалуйста, так ли это ? Я так же потерялся в вопросе, делает ли квантовая случайность непознаваемым наш мир ?
29 456358
>>56352
И что дальше? Практического применения у гравитонов нет.
30 456360
>>56357

>познаваемым


Ты ответь сначала хотя бы для себя - что есть познание?
Без имени2341 Кб, 1440x1440
31 456362
Вчера (сегодня) в час ночи пришла идея о мире-кольце ввиде самозамкнутого эээ сверла/спирали. Вращение как на пике по идее должно обеспечивать плавную смену времени суток и какую-никакую имитацию притяжения на 50% площади (по половине с каждой стороны). Но жопа чует какой-то подвох? Какой?
32 456366
>>56356
Причём тут хорошо или плохо, если вопрос про экономику?
Мне кажется что произойдёт просто закономерный рост цен и всё, эффект аннулируется. Я нигде внятного объяснения почему этого не должно случиться не видел. Единственный положительный кратковременный эффект который можно придумать это что-то кейнсианское, типа люди поднявшиеся из нищеты перестанут трястись за каждую копейку и сберегать и начнут больше тратить. Некоторые может и предпринимательством займутся. Опять же, правый аргумент в том что всякое институциональное гадство типа пенсий и велферов можно будет отменить и поразгонять соответствующие департаменты.
33 456367
>>56366

>Мне кажется что произойдёт просто закономерный рост цен и всё, эффект аннулируется.


Хммм...
А слона-то я и не приметил, как говорится. Да, самая очевидная вещь почему-то ускользнула от меня. Доходы населения вырастут, вырастет спрос, а следом и цены. Да.
Я-то уж думал, что все пропала надежда на построение коммунизма.
34 456368
>>56196 (OP)
Мы срздали жинь на основе кислорода и послали спутник за пределы солнечной системы.
А могли ли мы создать такую жизнь не на основе кислорода?
35 456369
>>56368

>Жизнь на основе кислорода


Чиво блядь?
37 456374
>>56368
Этот упоролся, несите нового.
38 456380
>>56196 (OP)
Максимально тупой вопрос, за который меня умные люди оплюют, ибо студентота недоразвитая: дали мне задание, ни разу не домашнее (честно, мне интересно стало, вопросы задавал лишние - вот и нарвался): "решить 1-д уравнение Клейна-Гордона при начальных условиях, а также "найти эволюцию решения с нулевой начальной скоростью и начальными условиями в форме прямоугольного импульса"

Вопрос даже не в том, что делать. Вопрос в том, что подразумевается под "найти эволюцию решения" и что такое "начальные условия в форме прямоугольного импульса"...
----.jpeg39 Кб, 446x314
39 456382
Существуют ли опубликованные результаты полноценных опытов по тренировке памяти у здоровых людей? Не та хуйня про то, как три бабы васи и двa бомжа с улицы три раза попытались воспроизвести последовательность из 10 символов, а реальный дроч на память десятков адекватных добровольцев на протяжении как минимум года?
40 456386
Есть два генетически абсолютно идентичных человека, до пяти лет их воспитывали одинаково, то бишь личность у обоих в порядке, но один филонил всю жизнь до 30 лет, учась новому на похуй, а второй усиленно зубрил и всесторонне саморазвивался как только мог. Интересует мнение по Савельеву на счет соотношения качества их мозгов: будет ли тот "близнец", что дрочил науки обладать более массивным и продуктивным по части мышления мозгом, или мозги будут идентичны, просто у "умника" будет больше инфы записано на "харде"?
41 456387
Вопрос от гуманитария... Вот, грубо говоря есть Планковская длина, а есть обратная величина с +?
42 456389
Бля челы срочно надо видос(ссылочку). Там сидел мужик, клацал по гугл картам и говорил мифе численности землян, типа -" вот посмотрите, на остановке 3 человека если бы нас было 7 миллиардов то тут было бы 500"
280227.jpg50 Кб, 600x450
43 456393
Поясните гуманитарию. Теория хаоса гласит, что малейшее изменение в системе может привести к абсолютно непредвиденным последствиям. Скорее всего,таких изменений во вселенной с начала большего взрыва было очень много. Не ошибочны ли наши суждения о большем взрыве, ведь между ним и нашим временем слишком много "искажений" ?
44 456394
>>56393

>Теория хаоса гласит, что малейшее изменение в системе может привести к абсолютно непредвиденным последствиям.


И где такую хуету читают.
Yoba3.jpg142 Кб, 600x600
45 456396
>>56394
Смотрят. Документалка BBC - The Secret Life of Chaos
46 456397
>>56396
Ужас какой!
47 456398
>>56389
Ищи в ютубе «Абдуль теория завышенной численности». Как то так, надеюсь, ты ещё увидишь. Кстати, он сейчас выпускает контент под ником «Внук Елькина», если вдруг интересно станет.
48 456399
>>56393
Вопрос только в том, является ли такая система хаотичной или нет.
49 456404
>>56387
Если основная теории сейчас то что вселенная бесконечна-то нет.
50 456405
>>56382
Ты говно жрать хочешь?
51 456408
>>56398
Спасибо большое!!!!
52 456442
Если вселенная бесконечна, то очевидно, что существует ненулевое количество планет, где развитие шло ровно так же, как на Земле, за исключением того, что сейчас анон >>56404 берет за щеку у анона >>56408 ?
53 456447
>>56442

>ненулевое


Бесконечное.
54 456451
d
55 456473
Каким образом в человеке вшитые природой поведенческие алгоритмы сочетаются со свободой мышления, которая влечёт способность "произвольно" менять нейронные связи в мозгу? Вот, например, говорят в человеке крепко вшита социальная установка делить всех на своих и чужих и нихуя с этим не сделаешь,- каким образом она так намертво прописана в мозгу? И в каком возрасте проявляется? В момент рождения что ли?
56 456489
Насколько хуёво было бы сейчас биосфере, если б на ядерные отходы всем было похуй, тупо бы их в лес или в море топили?
57 456493
>>56489
Нинасколько, в Чернобыле всё цветёт и пахнет только так.
58 456500
Мне одному кажется, что если сравнить технологический прогресс с 1900 по 1919 года и с 2000 по 2019 разница выходит не совсем в пользу второго?
59 456501
60 456504
>>56501
Красивая идея. Насколько правдоподобна - хуй пойми
Что-то сродни идее языка-инстинкта по фантастичности
61 456521
>>56270
Колебания водорода тоже можно дискретизировать как-то на двоичные данные. Чисто от балды предположим - амплитуда дает 8 четко различимых позиций, частота - еще 16. Итого в одном колебании 128 бит можно закодировать. Битами оперировать действительно удобнее всего, поэтому вместо 128-разрядного компьютера будет все равно битовый.
62 456522
>>56374
Чет вголосину
63 456523
>>56442
Это самая поразительная мысль, которую я читал в этом году. Серьёзно. Бесконечность, оказывается, страшная штука.
64 456524
>>56447
Может даже и нулевое быть, на всех остальных, например, точно так же, как на нашей, все происходит.
65 456525
>>56500
Ну не знаю. 8К телики вот вышли уже по 98 дюймов с практически бесконечно большой контрастностью. VR. Мощь суперкомпьютеров 80х в ладони, без проводов, весом 200 г, с интернетом несколько сот мегабит в секунду (столько же на весь африканский континент было поди в 2000 году). Комбинированные сенсоры Red Helium, могущие снимать в охуительнейшей четкости с кристальной чистотой 12К видео. Антиретровирусный коктейль против ВИЧ, антивирус против Гепатита С.
66 456526
67 456532
Может кто знает в каком научпоп видосе я мог слышать фразу о том, что дальнейшее экспериментальное углубление изучения структуры матереии стало невозможно, пока что, и на очень длительный срок, т.к. чтобы двигаться/пробовать расщеплять дальше нужны энергии другого порядка, недостижимые на земле/в установке, которую возможно было бы разместить на земле.
Смотрю висос, виритасиум, и то что ютуб в рекомендации от них кинуть может, видосик вроде был заграничный (переведённый на русский).

Всплыло в памяти, попытался найти и не вышло.
68 456533
>>56493

>Нинасколько, в Чернобыле всё цветёт и пахнет только так.


Так чё, реально то что в одном видосе говорили, что от чернобыля погибли только те кто в слоновью ногу наступил прям урановые брызги застывшие голыми руками носил и никакого всплеска онкологических заболеваний и уменьшение времени жизни тех кто под совсем прямое влияние его(урановые брызги) не попал-выявленно небыло?
Мне пиздежом показалось.
мимо другой, любитель видосиков
69 456539
Посоветуйте микроскоп для дома.
70 456540
>>56539
Хочешь на собственный член посмотреть? Цели какие?
71 456542
>>56540
Приобщиться к биологии/химии. Увидеть бактерии, микроорганизмы, структуру материалов в реальности.

А вообще была идейка сделать спермограмму своими руками =)
72 456556
Поясни, двач.
Если E=mc², то это зависимость энергии только от массы или и от скорости света тоже? Но скорость света всегда одинакова.
73 456557
>>56556
А что тебя смущает?
74 456558
>>56556
Онли от массы.
IMG6826.jpg136 Кб, 1200x1200
Молния-Маккуин 75 456565
Умные ребята, прошу ответить на шесть наитупейших вопросов касательно молнии и вообще явлений электрической природы, никак не могу вразумить (держите гуманитария!).
1) Может ли молния попасть в незаземленный объект? Например в птицу, подброшенный вверх мяч, самолет, или в предмет, лежащий на вершине деревянного строения.
2) Если да, то какая разница между попаданием в заземленный и незаземленный объект? Громоотводы ведь для чего-то существуют, получается, что если заземлено, то молния без происшествий уйдет в землю, а если плохо заземлено, то будет бабах и разрушения? Почему этот кабель и антенна громоотвода не разлетаются на кусочки, а камни или деревья разлетаются?
3) Может ли молния попасть в движущийся объект?
4) Почему зимой нету молний? Слишком мало влаги в атмосфере?
5) По каким причинам молния бьет только с неба в землю, а не с земли вверх? (наверное это самое тупое что я спрашивал в жизни)
6) Ветер с метелью вызывает напряжение статического поля у предметов и строений, а простой ветер вызывает? Без снега то есть. Если я феном долго буду дуть на что-то, а потом дотронусь, меня же щёлкнет?
76 456566
Хочу достигнуть глубокого понимания физики.
Ландау Лифшиц пойдёт?
77 456567
>>56556

>Если E=mc²


Во-первых, E(0)=mc², это для покоящегося тела, с - константа. Во-вторых, более полное уравнение учитывает импульс: Е²=(mc²)² + p²c²
78 456569
>>56566

>Хочу достигнуть глубокого понимания физики


в ютубе ищи по ключевым словам - научный трибунал
79 456585
>>56566
Только выработать в себе некоторую интуицию, а для этого тебе нужно хорошо отточить владение соответствующими математическими методами.
В идеале ты должен дрочить задачи и проводить по ним всякие опыты для закрепления. В ВУЗах не зря проводят лабораторные работы.
В любом случае, сначала ты должен на достаточно хорошем уровне знать матанализ и линейную алгебру, без этого совсем никак, потому что сразу за ними идут диффуры, вариационные счисления, теория групп и функциональщина. Чем глубже в физику, тем злее там математика. Тот же ОТО требует знания дифференциальной геометрии и тензорного анализа.
ЛЛ это во первых теоретическая физика, в ней все явления обобщаются до совсем голых моделей. И по умолчанию там считается, что ты хорошо знаком с обобщаемыми явлениями. Во вторых в ЛЛ упущен матаппарат, в нем часто выбрасываются дохуя выкладок и обходятся многие важные теоремы.
Перед ЛЛ рекомендую почитать мат методы и другие работы Арнольда, если неосилил, то ЛЛ можешь даже и не пытаться.
80 456594
>>56196 (OP)
какая физическая величина может измеряться в единицах Кл^2/Дж?
81 456595
>>56585
А если я поступаю в годный физический вуз (там с олимпиадкой повезло как-то), то что посоветуешь, чтобы на первом курсе было не совсем жёстко?
82 456598
>>56595
Дрочи учебу и не еблань.
Программа ВТУЗов по физмату не сильно различается, один хуй тебе все нужное дадут преподаватели.
83 456600
>>56565
1) да может
2) разницы нет
3)да
4)вот тут нет пока идей
5)молния - разновидность движения зарядов т.е они идут от того места где они есть туда где их нет
6) нет ,причина подобного явления у метели трение и татические электричество
84 456602
>>56447

> >ненулевое


> Бесконечное


Да, но это настолько дикое слово в данном контексте, что я побоялся его использовать
>>56442-кун*
а еще где-то я прямо тебя в очко ебу :3
85 456604
>>56565

>По каким причинам молния бьет только с неба в землю, а не с земли вверх?


А она туда-сюда и бьет, вот сюрприз, мерцает при ударе не просто так. Молния это не так тривиально, как кажется.
Вот для примера: https://www.youtube.com/watch?v=koI4dGUBS_8&t=1s
86 456607
>>56600
Почему дерево разрывает в щепки при ударе молнии?
мимо
87 456612
>>56607
Очевидно из-за огромных токов происходит резкий нагрев до высоких температур, и в следствии сопутствующих процессов древесина просто взрывается.
88 456637
Поясните наконец, разная скорость метаболизма - это миф или нет? Типа кто-то может есть как не в себя и не толстеть, а кто-то от одной булочки разбухает. В физаче, например, за такие слова ссут на лицо, а вы что думаете?
89 456645
>>56567
Является ли формула e=mc² частным случаем формулы импульса p=mv, где p - импульс, m - масса и v - скорость?
90 456646
>>56637
Хз, вот я ем очень много. И постоянно сладкое. Могу за день съесть 3-4 шоколадки, коробку зефира, запить молочным коктейлем а под вечер навернуть 2 йогурта и какао. У меня наверное зависимость какая-то от сахара (диабета нет).
В принципе, это и есть мой обычный дневной рацион.
Ни намека на лишний вес, спортом не занимаюсь. Меня ещё спрашивают, почему я такой худой.
Чувствую себя прекрасно.
91 456647
>>56637

>от одной булочки разбухает


Это точно нет, т.к. закон сохранения массы.
Просто жирные проглоты пиздят и давят на жалость, а сами только и делают что жрут.
А вот с первым-возможны варианты что хуже усвоение веществ у худых, и они просто высирают почти непереработанные продукты. Но я тут хз.
Хотя других вариантов и нет. Если он не пиздец дохера двигается-то только в температуру могла бы уходить масса/энергия, а она у них как и у всех 36.6, тут всё легко проверяется.
92 456648
Насколько востребовано в технике было бы золото, если б его было дохуя, хотя б как меди?
93 456650
Под солями.
94 456651
95 456652
>>56648
Ну если как меди, то востребованнее чем медь, ведь у него лучше свойства при той же экономике (редкости) по такому раскладу.
96 456662
>>56645
странный вопрос, слагаемое mc² отображает энергию покоя, pc энергию движения. Для света Е=pc.
97 456663
>>56647

>т.к. закон сохранения массы


это в какой науке такой закон?
98 456675
>>56663
Это в химии
мимо
99 456676
>>56675
Физике...
100 456684
>>56645
Нет, mc2(скаляр) вылезает в лагранжиане для свободной частицы, двигающийся в пространстве минковского.
p=mv(вектор) вылезает из лагранжиана только при специальном преобразовании для частиц двигающийся в евклидовом пространстве.
Чем замечательная теория поля(СТО), что время и пространство обедняются в одну сущность, что позволяет убрать нахуй разнородные преобразования, правда получаемая величина становится уже не скаляр/вектор, а тензор импульса-энергии, да еще в добавок симметричный по определению. Величина m2c2 постоянный (неизменяемый при преобразование вроде смены системы отсчёта или других преобразований) результат "сокращения/упрощения" тензора импульса-энергии. Из нее и растут ноги ваших знаменитых формул.
101 456689
>>56637
Для запасания жира в липоцитах должен пройти целый каскад реакций с кучей регуляторных белков и ферментов, который ещё и идёт параллельно переводу глюкозы в гликоген в печени.
Т.о. есть люди склонные к ожирению и есть не склонные к ожирению, что генетически обусловлено. С более активными липоцитами, или обилием жирозапасающих гормонов – жиреют быстрее.
С другой стороны, у человека есть определённый минимум сжигаемых калорий в сутки, который не зависит от того, отдыхает ли он, или трудится тяжело (но не убивается как спортсмен, или не под кратковременным стрессом как солдат в бою). Так, городской житель-водятел сжигает в среднем примерно столько же калорий, сколько и охотник-собиратель, передвигающийся пешком,
просто у охотников обмен веществ оптимизирован.
В общем-то дело не в скорости метаболизма, а в балансе калорий на те или иные процессы, и активности жироотложения
102 456691
>>56637
То, как о метаболизме говорят жирухи-дрищи это стопроцентный миф. Так что правильно обсывают.

Склонных к жирноте от природы не бывает. Но бывают склонные к образу жизни, которые приводит к ожирению. Тут тоже гормоны играют не малую роль, но они в первую очередь влияют именно на образ жизни и аппетит, а не на твой метаболизм.

Допустим тяньки любят говорить о том, что жиреют от гормонов. Но это не так. Жиреют они от того, что их метаболизм наоборот может замедлится, это проявляется в том, что они становятся более вялыми, больше спят, но при этом едят столько же, а то и больше, ибо аппетит может повыситься. То есть она стала меньше энергии тратить, но жрать столько же или даже больше, а значит будет толстеть.

Тяньки вообще более склонные к ожирению именно из-за образа жизни в первую очередь. У них метаболизм медленнее мужского + они сами меньше, а в семье она может есть с ним на равных. И это гарантировано приведёт как минимум +30кг лишнего веса у неё, когда он будет более менее стройный.
103 456695
В научпопах про маглев пишут, что по теореме Ирншоу нельзя сделоть левитацию (в поле тяготения Земли) на основе только постоянных магнитов. То есть, даже если всю Землю утыкать магнитами северным полюсом вверх, и сделать квадратную платформу 1км*1км и утыкать ее магнитами северным полюсом вниз, левитации не произойдет? А что случится? Платформа перевернется и прилипнет к земле южными полюсами магнитов?

Предполагается, что сила магнитов такова, что они могли бы левитировать платформу по крайней мере в неустойчивом равновесии (то есть если бы ее кто-то слегка пальцем удерживал от переворота).
104 456698
>>56556
Одно и тоже.Из года в год.

>Если E=mc², то это зависимость энергии только от массы или и от скорости света тоже? Но скорость света всегда одинакова.


Нет, нет и еще раз нет. Правильная запись этой формулы: E_{0} = mc^{2}. Соответственно нет никаких массы зависящих от скорости и прочей белеберды.
105 456699
>>56645

>Является ли формула e=mc² частным случаем формулы импульса p=mv


Нет не является. Она являтся частным случаем формулы Е²=(mc²)² + p²c²
106 456741
Если взять новорождённого младенца и прибить ему к голове VR очки, через которые показывать четырёхмерное пространство, мозг адаптируется на то, чтобы в нём ориентироваться? Другими словами, то что у нас мозг заточен на 3д, это врождённое или приобретённое?
ЗдраствуйДерево.jpg86 Кб, 645x729
107 456743
>>56741

> прибить ему к голове VR очки, через которые показывать четырёхмерное пространство


Для восприятия четырехмерного пространства нужна как минимум трехмерная сетчатка (то есть сорт оф трехмерный массив палочек@колбочек).
108 456756
>>56743

>Для восприятия четырехмерного пространства нужна как минимум трехмерная сетчатка


Что мешает проецировать 4д на 2д экран?
109 456781
Пары плюмбума и образование оксидов свинца из сплавов для припоев. В основном с оловом и малыми количествами меди.
Часто встречаю в интернете сведения, что при пайке оловянно-свинцовыми припоями свинец испаряется. Допустим, что температура пайки чаще всего не превышает 380°C.
Это верные данные, про испарение свинца?
110 456789
>>56756
нарисуй 3д на одномерной плоскости. если получиться, то сможешь проецировать и 4д на 2д.
111 456797
>>56781
Да, свинец, как любой другой метал испаряется. Только вот скорость испарения довольно низка, ОЧЕНЬ низкая.
При 600 К давление паров 59,6 Па. Будет ждать месяцев/лет.
Другое дело, что свинец может выходить из припоя и растворяться в проводнике, а еще свинец из-за химических процессов может портить качество припоя.
112 456804
>>56797
Наверное вопрос неправильно задал.
Имел ввиду, что свинец реально испаряется при нагреве жала паяльника ~380°C, что есть опасность вдыхания паров уже при комнатной температуре и накопления его в организме человека?
113 456827
>>56804
В принципе да, если упорно дыщать парами свинца, можно заработать отравление им.
Однако, свинец не такой токсичный как кадмий или ртуть. Свинец плохо проходит липидные мебраны, и плохо цепляется за их поверхность.
При комнатной температуре давление паров свинца на уровне миркоПаскалей, да еще свинец покрывается своем оксидов, из-за чего можно считать не испаряется.
114 456847
>>56789

>на одномерной плоскости


На геометрической прямой линии, лучше сказать.
115 456872
Вопрос по теории управления/теории автоматического регулирования систем. Я никак не могу въехать в эту дисциплину, ищу учебник для чайников. В большинстве просмотренных мною книг просто последовательно вводятся понятия передаточной функции, ЛЧХ, критерии устойчивости и т. д. Но я не могу это понять пока не понимаю зачем это нужно. Т. е. с какой целью данные понятия вообще были изобретеныи введены в теорию управления. Я как маленький ребёнок: в дестве я не понимал как на физике объекты описывают с помощью цифр. Но школа эту пробелму решила. Я понял для чего вводятся те или иные характеристики в физике. А в теоии управления вот не могу понять потому что их просто вводят но не объясняют для какой собственно задачи это нужно. Не могу связать начало и конец, сколько не изучаю литературу.
116 456917
>>56872
Этого всего не понятно из определений этих понятий?
800px-FuelcellNASAp48600ac.jpg176 Кб, 800x1017
117 456924
А что там насчет топливных элементов, аноны? Почему мы до сих пор не засовываем их в автомобили и мобильные устройства? Какой топливный элемент проще всего сделать дома на кухне из говна и веника? Какие перспективы развития топливных элементов? Где вообще читать инфу о них и общаться с мамкиными Кулибиными, есть какие-то ресурсы?
118 456927
Если проводить аналогии между электричеством и движением жидкости, и соответственно между проводами и шлангами, то с пропускной способностью по объёмы жидкости/электричество шлангов/проводов всё понятно.
А что насчёт аналогии прочности шлангов по давлению относительно электричества?

Существует такое напряжение, при котором материал провода просто разорвёт или типа того?
119 456937
>>56927
Существует сила тока, при которой провод может расплавиться. Например, если вместо сварочного кабеля попытаться варить проводом от настольной лампы, то он расплавится. То есть, для высоких токов нужен провод бОльшего сечения.
120 456940
>>56937
Я это знаю, в вопросе это отражено, я спрашивал про давление/напряжение.
121 456941
>>56940

>Я это знаю, в вопросе это отражено, я спрашивал про давление/напряжение


Намекаю тебе I=U/R
122 456942
>>56941
Ты не понял вопрос, так что оставь свои намёки при себе.
3.gif4,1 Мб, 400x200
123 456948
>>56942

>Ты не понял вопрос

124 456966
>>56948
Ты действительно не понял вопрос, анончик спрашивает про пробой изоляции.
125 456969
>>56948
Да да, заглушку в свою формулку подставь и нагнетай бошьше и больше давления.
126 456970
>>56966
Я думал над этим, но нет, не то.
Может ли при повышении напряжение в материале проводника начнут увеличиваться какие-то силы, которые в какой-то момент привысят силу межатомного притяжения?
127 456971
>>56966
>>56970

>Я думал над этим, но нет, не то.


Нет) Он спросил не про это.
128 456977
>>56927

>Существует такое напряжение, при котором материал провода просто разорвёт или типа того?


в смысле взорвется без нагрева? Нет. При большой плотности тока провод расплавиться или частично испарится, со взрывом, такое бывает при коротком замыкании.
129 456980
>>56977
Да что ты мне про этот йобаный ток снова и снова, знаю я что ток вызывает нагрев и знаю что от него провода могут хоть расплавиться хоть испариться.
Энергия получается из произведения тока на напряжения, нельзя запихнуть в конечный объём бесконечную энергию или передать бесконечную энергию по проводнику конечного сечения.
Оставляя ток ниже предела который вызывает расплавление провода можно увеличивать энергию увеличивая напряжение.
До каких пор?
130 456984
>>56980

>Энергия получается из произведения тока на напряжения


это мощность которая в свою очередь энергия в единицу времени

>Оставляя ток ниже предела который вызывает расплавление провода можно увеличивать энергию увеличивая напряжение


закон Ома игнорируется?
131 456996
>>56984

>закон Ома игнорируется?


Ну йобана, у тебя кроме проводов ничего нет и ты решил их замкнуть сразу?
Поставь нагрузку с сопротивлением, при котором ток не будет превышать тот, который расплавит провода, и повышай напряжение.
132 457000
с точки зрения безопасности, опаснее не соединять ванну в ванной комнате с другими электропроводящими устройствами/поверхностями?

я так смотрю на задачу.
есть три железные поверхности.
1. кран воды
2. железная ванна
3. батарея отопления или стиральная машинка.

между краном и батареей есть разность электрических потенциалов достаточная для того, что бы убить.

А) случай когда ванная, батарея и кран соединены.
в этом случае вероятность того, что отпадет одно из соединений и человек возьмется за поверхность от которой отпало соединение:
P_12 + P_23
индексы, 12, 23, это соединение между 1 и 2, 2 и 3.
это вероятности смерти.

Б) случай когда ванна не соединена с краном и батареей.
P = P_12` P_23` --- умножаются
где
P_n` -- вероятность касания поверхностей с индексами.
P -- вероятность касания обоих поверхностей - вероятность смерти.

использую вероятности введенные в Б, перепишем для А:
P_12 = P_12` P_12б --- умножаются
P_23 = P_23` P_23б
где
P_nб это вероятность обрыва соединения между поверхностями обозначенными индексом n.

нам нужно сравнить: P и P_12 + P_23.

представим, что кран и ванна соединены. душ валяется в ванной
P_12` = 1
тогда
P = P_23` -- смерть зависит от вероятности дотронуться до батареи.
и
так-как P_12` = 1 => P_12б = 0 => P_12 = 0
P_12 + P_23 -> P_23` P_23б --- смерть зависит от вероятности разрыва соединения с батареей и одновременно, касания батареи человеком.

P_23` < 1 и P_23б < 1 => P > P_23` P_23б
получается, безопаснее когда батарея и кран, соединены.

или нет?

но нужно бы рассмотреть соединение проводом ванны и крана и соединение ванны и крана душем по отдельности.
132 457000
с точки зрения безопасности, опаснее не соединять ванну в ванной комнате с другими электропроводящими устройствами/поверхностями?

я так смотрю на задачу.
есть три железные поверхности.
1. кран воды
2. железная ванна
3. батарея отопления или стиральная машинка.

между краном и батареей есть разность электрических потенциалов достаточная для того, что бы убить.

А) случай когда ванная, батарея и кран соединены.
в этом случае вероятность того, что отпадет одно из соединений и человек возьмется за поверхность от которой отпало соединение:
P_12 + P_23
индексы, 12, 23, это соединение между 1 и 2, 2 и 3.
это вероятности смерти.

Б) случай когда ванна не соединена с краном и батареей.
P = P_12` P_23` --- умножаются
где
P_n` -- вероятность касания поверхностей с индексами.
P -- вероятность касания обоих поверхностей - вероятность смерти.

использую вероятности введенные в Б, перепишем для А:
P_12 = P_12` P_12б --- умножаются
P_23 = P_23` P_23б
где
P_nб это вероятность обрыва соединения между поверхностями обозначенными индексом n.

нам нужно сравнить: P и P_12 + P_23.

представим, что кран и ванна соединены. душ валяется в ванной
P_12` = 1
тогда
P = P_23` -- смерть зависит от вероятности дотронуться до батареи.
и
так-как P_12` = 1 => P_12б = 0 => P_12 = 0
P_12 + P_23 -> P_23` P_23б --- смерть зависит от вероятности разрыва соединения с батареей и одновременно, касания батареи человеком.

P_23` < 1 и P_23б < 1 => P > P_23` P_23б
получается, безопаснее когда батарея и кран, соединены.

или нет?

но нужно бы рассмотреть соединение проводом ванны и крана и соединение ванны и крана душем по отдельности.
133 457008
>>56996
Тогда очевидно рано или поздно пробьет через воздух, если у него сопротивление окажется меньше.
134 457011
>>57000

>между краном и батареей есть разность электрических потенциалов достаточная для того, что бы убить.


Охуеть, откуда?
135 457016
>>57011
в многоквартирных домах трубы отопления, трубы холодной воды, должны быть соединены и заземлены. но не проверять же каждого жильца? он может заменить кусок трубы на трубу из изолятора, так-как дешевле. другой житель, заземлит электроприбор на трубу.
или вместо радиатора батареи, рядом с ванной стоит стиральная машинка у которой на корпус отпал провод из розетки.
136 457030
>>57016
И с каких это пор у нас в розетке достаточный для убийства потенциал, лел?
137 457038
https://www.youtube.com/watch?v=pC9DK-vdiXY
Возник вопрос - он ведь специально пиздит, а не просто дегенерат?
138 457057
>>57030
сопротивление кожи и ток фибрилляции:
https://www.wikilectures.eu/w/ELECTRICITY_AND_HUMAN_BODY

нужно от 60 мА для фибрилляции.
у мокрой кожи сопротивление 500 ом на 1 см^2.

у руки длиной метр и длиной окружности 17 см, сопротивление примерно 100 500 ом/17 = 3 ком. 220/3000 = 73 мА.
139 457069
>>57038

>наука зашла в тупик


Ой, всё.
140 457231
Внимание, вопрос: если в нейтронной звезде гравитация загонят электроны в протоны, то почему они становятся нейтроном, если электрон не составная частица и в нем нет кварков?
15335774501120.jpg36 Кб, 634x402
141 457243
Пачаны, а как собираются снимать энергию с термоядерных реакторов? Опять будут воду выпаривать, чтоб турбину с генератором крутить?
143 457247
Посоны, объясните. Допустим у нас есть рация с квантовой связью на ЭПР-парадоксе. Допустим у нас есть специально оборудованное транспортное средство ну или, если хотите летательный аппарат, которое способно разгонятся до таких скоростей, что человеку становятся заметны эффекты СТО, без вреда для пассажира. Эксперимент заключается в следующем: Человек на корабле разгоняется до скоростей, при которых нам, находящимся в условном покое, кажется что время в его системе отсчета замедленно в 2 раза, он проговаривает в квантовую рацию фразу условной длины длинной пусть в 3 секунды по его часам. Как мы её услышим?
144 457248
>>57247

>рация с квантовой связью на ЭПР-парадоксе


Зашел с козырей я смотрю. Передача информации с помощью квантов запрещена.

>Как мы её услышим?


Если предположить что у нас есть подобный предмет, то в два раза медленнее будем слышать.
145 457266
Сильно ли от апноэ деградируют те клетки мозга, которые через 5 минут без кислорода помирают?
146 457267
>>57248

>Передача информации с помощью квантов запрещена.


Кем? Галактической конфедерацией? Почему нельзя?
147 457288
>>57267
Да много кем, там целый лагерь уч0ных.
Нет, принимали решение несколько десятков лет.
Строгий ответ кроется в теории групп. Точнее когда теорию групп пытались применить к квантовой теорию, обнаружили элегантное закрытие проблемы.
148 457293
Предположим, мультивселенная бесконечна, вселенных бесконечно много, а значит найдется полная копия нашей, с такой же землей, двачем и тобой, анон. Эту мысль часто произносят в научпопе.

С другой стороны, бесконечность подразумевает бесконечное количество вариантов и отклонений, вариативность бесконечно неисчерпаема.

Вопрос — найдется ли в бесконечной мультивселенной ТОЧНАЯ копия нашей вселенной или нет?
149 457294
>>57243
>>57243

>Опять будут воду выпаривать, чтоб турбину с генератором крутить?


Критикуешь — предлагай
150 457295
Натуральные числа формируют числовой "луч".
Вещественные — числовую "прямую".
Комплексные числа формируют уже "плоскость".

Какой уровень следующий? Какие числа образуют числовое "пространство" (3 мерности)?
151 457297
>>57295
Комплексные числа это один из примеров расширения вещественных чисел до гиперкомплексных. Так же там есть всякие кватернионы, октонионы, седенионы. Трехмерности они не образуют - все идет по степеням 2.
152 457299
>>57295
Вообще этим занимается алгебра.
Ты имеешь ввиду, какие алгебраические структуры можно сопоставить "пространствам" высших размерностях?
Или какие "пространств"а можно сопоставить определенным алгебраическим структурам?

Вообще любое линейное пространство является группой.
Но не любое линейное пространство уже будет обладать свойствами как вещественные и комплексные числа, т.е на них нельзя будет задать нужное умножения.
153 457308
>>57299
когда узнал что комплексные расположены по оси Х и образуют вместе с вещественными некую "плоскость", стало интересно что дальше, какие числа идут по Z и образуют "объем" чисел

как выяснилось — гиперкомплексные
154 457309
комплексные по Y — быстрофикс
155 457310
>>57308
Расширять вещественные числа можно вообще произвольным образом(приделывай сколько угодно дополнительных измерения), другое дело в определение операции умножения и как она вяжется со сложением. К примеру, можно на трехмерных векторах определить операцию векторного произведения, только она не коммутативна и содержит делители нуля. Есть специальная теорема, где доказывается, что расширять числа с привычным нам умножением дальше нельзя.

Впрочем числа можно расширять в "другую сторону", к примеру расширение вещественных чисел в пространство функций. Оно бесконечномерное и нем появляется другие операции, схожие с умножением и делением.
156 457315
>>57295
гиперкомплексные кватернионы, кстати, позволяют очень быстро вращать объекты в пространстве. Там достаточно складывать и перемножать, тогда как в обычном пространстве надо находить синусы углов. Потому в игорях вся графика, прежде всего рассеяние света, увязана на гиперкомплексные вычисления.
157 457318
>>56358
А ещё его не было у радиоволн.
158 457320
>>56357
А я агностик. Наш мир непознаваем потому что максимум на что мы способны заперто внутри черепной коробки и при том все каналы связи с внешним миром у нас крайне скудны, и мы физически не можем познать никаких аспектов реальности, изначально недоступных нам, подобно тому, как слепой от рождения не может понять, что такое цвет.
159 457321
>>56380

>вопрос не в том как это сделать а хотя бы в том что блин значат эти непонятные слова...

160 457324
>>57293
Найдется бесконечное количество точных копий.
7249039[1].png317 Кб, 1002x626
161 457327
>>57320

>А я агностик


Пикрилейтед

>Наш мир непознаваем потому что максимум на что мы способны заперто внутри черепной коробки и при том все каналы связи с внешним миром у нас крайне скудны, и мы физически не можем познать никаких аспектов реальности, изначально недоступных нам, подобно тому, как слепой от рождения не может понять, что такое цвет.


Если "твой" мир не возможно познать, то каким образом ты общаешься со мной если мы друг-друга не видим и даже не знаем? Я кстати сейчас являюсь тем самым аспектом реальности, который якобы тебе недоступен.
15477033716690.png335 Кб, 1002x626
162 457340
>>57327
Агностицизм не подразумевает никакого отношения к богу. Это позиция принципиальной непознаваемости мира. Как все верующие являются агностиками, так и атеистам такой взгляд не противоречит.

В следующем абзаце ты вообще какую-то невнятную хуйню спорол.
163 457344
>>57340

>Это позиция принципиальной непознаваемости мира


Если мир не возможно познать, то почему китайцы смогли отправить анальный зонд на обратную сторону Луны? Почему возможен интернет? Почему есть акробаты знающие возможности собственного тела? Почему собаку поводыря можно обучить и она будет помогать слепцам? Мир не познаваем - это маняпозиция маняфилософии.
164 457348
>>57344
И какой из твоих вопросов опровергает агностицизм?
Всё, что ты знаешь о мире, является твоим субъективным опытом, пропущенным через твои собственные призмы и фильтры, и так у всех. Таким образом, просто никак невозможно собрать объективную непредвзятую картину мира.

Мы даже зелёный цвет можем воспринимать по-разному и не догадываться об этом. Или звуковые вибрации. Откуда я знаю, что ты чувствуешь их так же, как чувствую я? При всём этом, никакая интерпретация никакого человека не соответствует тому, что есть на самом деле.

И это только самые фундаментальные аспекты, получаемые нами непосредственно из взаимодействующего с нами мира, что уж говорить о более сложных вещах, существующих исключительно в голове, и которые, по сути, у каждого человека формируются заново.
165 457350
>>57348
Чувак и как ты такой неуверенный живешь вообще? Как с кровати встаешь? Вот исламисты бошки режут, ученые-в-говне моченые коллайдеры собирают. А ты что?
166 457351
>>57350
А я живу прививанием этого ощущения безысходности и другим людям.
167 457354
Я слышал такую гипотезу, что работу группы нейронов, можно представить в виде единиц и нулей. Что-то типа "наличие импульса это 1, а отсутствие 0"
Насколько это ахинея?
168 457355
>>57354
А как ещё оно может работать если не так?
169 457356
>>57354
Чем то что делает компьютер отличается от того что делаешь ты?
170 457366
Не понятно, как из этого обоснования закона сохранения энергии следует что энергия одинакова в любые два момента времени, а не только в такие, когда сохраняются координаты и скорости?
171 457367
>>57366
Тебе на каком уровне объяснить? По пике это уровень школьника или первокура.

Что такое интеграл движения без введение в лагранживый формализм, я считаю бессмысленным.
172 457370
Видели проект нового адронного коллайдера?
С его мощностью можно будет расщеплять соединения кварков?
Какие энергии нужны для расщепления кварков?
чего мы не знаем для построения теории всего?
173 457372
>>57366
Пиздец, поубивал бы таких "авторов". Ничего не доказал, только запутал, зато поумничал.
174 457385
Начал изучать биологию по трёхтомнику Тейлора. К сожалению он старого издания (1990). Где бы найти более современное?
Накопилось несколько вопросов:
1. Может ли бактерия иметь одновременно капсулу и фотосинтетическую мембрану? Первая не помешает второй выполнять свою функцию?
2. Мезосомы — часть клетки или нет?
3. Как при половом размножении методом трансформации ДНК донора проходит через собственную клеточную стенку и клеточную стенку реципиента? Клеточная стенка ведь не пропускает крупные молекулы нуклеиновых кислот и обладает антигенными свойствами из-за содержания белков и полисахаридов.
4. Почему для лечения болезней, вызванных грамотрицательными бактериями рекомендуют пенициллин? Грамотрицательные бактерии же обладают липидным слоем, который придаёт устойчивость к пенициллину.
5. Сочетаются ли линейные ДНК с кольцевыми плазмидами? А кольцевые ДНК с линейными плазмидами?
6. Способен ли вирус к самосборке из одних только капсомеров?
7. Как вирус может проникнуть в клетку кроме как методом инъекции?
8. Пишут, что вирусы произошли от клеточных организмов, т. е. от нуклеиновой кислоты, которая приобрела возможность реплицироваться независимо от той клетки, из которой она возникла. Однако Докинз в «Эгоистичном гене» писал, что репликаторы-предки ДНК изначально имели возможность реплицироваться безо всяких клеток, а сама клетка — продукт дальнейшей эволюции. Где истина?
175 457405
>>57370

>Видели проект нового адронного коллайдера?


нет

>С его мощностью можно будет расщеплять соединения кварков?


Даже струны

>Какие энергии нужны для расщепления кварков?


10^28 эВ

>чего мы не знаем для построения теории всего?


взаимосвязи теорий с числом 42
sage 176 457406
>>57356
>>57355
Пиздец дегенератина. Мозг не работает по двоичной логике, ака процессор
1280px-DiagrammadiVenndeinumeri-ru.svg.png109 Кб, 1280x1280
177 457407
>>57310

Обычно матрешка числовых множеств заканчивается на комплексных. Кто может продлить иерархию? Какое место там замут супер- и гипердействительные, а также сюрреальные множества?
178 457408
>>57405
Очень, блять, смешно.
179 457412
Проверьте, пожалуйста, верны ли рассуждения.
---
В момент касания: N = -2Mg, Fтр = 2μMg.
Угол β = arctg(2μMg/Mg) = arctg 2μ.
---
Ответ получился верным. Но он был основан на расчёте сил, а не скоростей. Также непонятно, как обосновать, что N = -2Mg. Как это исправить?
180 457414
>>56309
ОНИ не дадут этого сделать, потому что это не в ИХ интересах. Смекаешь?
181 457417
>>57412
Тебе надо связать импульс и момент импульса.
182 457418
>>57412

>В момент касания: N = -2Mg


Нет, ты хоть раз ронял на ногу что-нибудь? Не казалось ли тебе, что сильнее бьет чем просто лежит?
183 457425
>>57407
Чую подвох в твоем вопросе, так что заранее не буду использовать некоторые термины.
Это твоя иерархия чисел чисто умозрительна. Расширение натуральных чисел(N) начали делать вполне для конкретных целей. У N определена операция сложения и так же возможно определить обратную ей операцию, но операция "вылазивает" за множество N, для этого множество расширили до Z. На этом множества, операция вычитания не выводит за предела множества.
Аналогично предстоит дело с рациональными числами Q. На Z нельзя определить обратную операцию умножения - деление. И тут проявляется множество Q. В нем для каждого элемента определена обратная операция умножения (кроме нуля).
Позже перешли к многочленам степеням. И там выяснилось, что обратная операции возведение в степень(взятие корня) так же не помещается в Q. Так придумали вещественные числа R. Впрочем с R связан момент, на нем есть непрерывность, которая позволяет этому множеству определяет новый тип операций - дифференцирование и связанный с ним элементарные функции. Об этом позже.
Оказалось, что не каждый многочлен имеет корень в R, так собственно и появились комплексные числа C. И когда запилили комплексный анализ, поняли, что все заданные операции на C, лежат в C. Расширяться дальше некуда.
Конечно были всякие попытки дополнительно расширять C, но выяснилось что это не дает профита. Дальнейшее расширение чисел только приводят только к кастрированной алгебрегруппам, а если дополнять какие-нибудь операциями, то они рано ли и поздно приведут к неполной алгебраической структуре на C - всякие линейные и топологические пространства.
184 457438
>>57425
Доходчиво!
Подвоха не было
Но я так и не понял вбирают ли сюрреальные числа в себя все остальные, включая всякие гипер, нано, супер или это отдельная иерархия, не пересекающаяся с иерархией пикрила выше?
185 457441
>>57412

>В момент касания: N = -2Mg, Fтр = 2μMg.


>Угол β = arctg(2μMg/Mg) = arctg 2μ.


>---


>Ответ получился верным


схуяли он верный? tg(β)=μ
186 457443
>>57438
Сюрреальные числа это метод построение множества, в некотором роде это "пространство" - множество на котором определена особенная структура. Расширением чисел трудновато назвать. Впрочем это в такое множество можно впихнуть множество реальных чисел. Но из R без задание структуру нельзя выйти в сюрреальныее.
Гиперчисла извращение пошедшее из нестандартного анализа. Пошло из того, что математикам не понравилось работать с пространством функций и решили ебашить производные алгебраически.

Короче, все эти дополнительные расширения получаются "расширениями в бок" относительно друг друга.
187 457445
>>57412
Сила реакции опоры растет с нуля до Nmax (которая больше чем обычное mg) потом падает до нуля опять. Сила трения аналогично рисует гору. Импульс тела по оси y меняется на величину интеграла от этой силы по времени. Импульс по оси x меняется на величину интеграла по времени от силы трения (этот интеграл просто равен предыдущему но умноженный на μ). А дальше уже рассуждения о упругости удара
188 457447
>>57443

>Пошло из того, что математикам не понравилось работать с пространством функций


Совсем нет.

>решили ебашить производные алгебраически.


И так ебашили, алгебры Ли и деривации это стандартные (pun not intended) инструменты.
189 457463
Не понимаю некоторые основы электротехники. Для чего нужен шунт. Пишут что подключив его параллельно нагрузке, можно уменьшить идущий через нее ток. Каким образом? Ведь сопротивление нагрузки постоянно, да и на напряжение шунт не влияет, так как подключается параллельно.
Далее, ещё читал что шунт включают параллельно с амперметром. И типа сумма токов шунта и амперметра равна току до разветвления. Но чем меньше сопротивление шунта, тем больше эта сумма, какое отношение она имеет к току который мы хотим измерить? Да и у амперметра же постоянное сопротивление, поэтому при данном напряжении ток через него должен идти одинаковый, что с шунтом что без, ничего не понимаю, как это работает.
190 457469
>>57463

>параллельно нагрузке, можно уменьшить идущий через нее ток. Каким образом?


Очевидно, что часть тока пойдет через шунт, т.к. это параллельное подключение.
191 457476
>>57445
Спасибо)
1543605391797.jpg121 Кб, 960x1280
192 457481
У меня вопрос про женские соски.
С биологической точки зрения зачем нужны сиськи млекопитающим самкам, особенно человеческим тянкам - понятно.
Соски тут выполняют роль, как я понял, губки-клапана, предоставляющей груднечку молоко, и не дающая как правило молока вне фазы вскармливания.
НО какой смысл в эрегировании сосков? При женском возбуждении у соска расширяется ареола и сама шишка соска напрягается и удлиняется. Зачем? Какой в этом смысл при сексуальном возбуждении? Я ещё понимаю, в чём суть эрегированного члена - чтобы воспроизвести бурение своего сверла в складки женской писечки до входа в матку, дабы совершить там впрыск плодородной жидкости. То есть практический смысл вставания шишки есть, а у соска я такового не вижу.
193 457483
>>57481
Для ребёнка - чтобы лучше молоко сосать. Для бабы - увеличение площади и, возможно, чувствительности эрогенной зоны (по аналогии с клитором).
1547447310991.gif1,9 Мб, 359x359
194 457487
>>57483

>Для бабы - увеличение площади и, возможно, чувствительности эрогенной зоны (по аналогии с клитором).


Ну и какая в том практическая цель?
195 457490
>>57487
При женском обрезании желание ебаться пропадает (да и ебать бревно нет удовольствия) => в среднем такие сельди сделают меньше личинок.

Значит, обратное - увеличение желания ебли за счет увеличения чувствительности эрозон - даст увеличение фертильности (среднего числа личинок у особи с данным признаком). А больше фертильность => признак генетически закрепится.
196 457491
>>57490
А что ты скажешь про короткостволов от мира тян - мелкосисечных узкососочниц? У них ареола не расширяется, максимум что сосок твёрже будет.

И вообще, я чувствую какую-то подмену понятий в твоём ответе - не коррелирует фертильность с сосками, скорее ёр аргумент из инвалид
197 457492
>>57487
Какая цель у удовольствия при сексе? Чтобы особи к нему стремились и он у них гладко и успешно проходил, очевидно.
198 457507
В чем смысл головки члена? Почему не просто конус, как у собак?
199 457508
>>57507
Гугли спермовые войны.
200 457542
Работает ли магнетронная пушка от канала Креосан? Не раз доводилось слышать, что всё это постановка, но может чуваки просто не хотят гробить свою технику, а сама пушка таки работает?
https://www.youtube.com/watch?v=j3SOpT9ohYw
201 457547
>>57542

>Дрищи в фольге


>Магнетронный супер-пупер гиперболоид направленного действия


Конечно!
202 457555
>>57547
Микроволновка работает на магии? Или излучение нельзя направлять?
203 457570
>>57481
C чего вы решили, что у всего и всегда должна быть цель в любой ситуации? Человек это набор костылей, который строился миллиарды лет, а не сконструированный каким то инженером организм. Просто соски строились из примерно той же ткани, что и клиток, от того и реагируют так же на сигналы. Ты вот в курсе что твои уши это бывшие жабры? И что некоторые особенности жабр они сохранили до с их пор?
204 457571
Объясните, пожалуйста, корпускулярное и волновое излучение это принципиально разные вещи или по сути одно и то же? сам в физике не шарю, когда-то услышал про корпускулярно-волновой дуализм и теперь, когда в каких-то книгах натыкаюсь на такое деление, каждый раз возникают мысли можно ли вообще так классифицировать
205 457572
>>57571
То и другое просто математические условности. Частицы ведут себя и как волны, и как отдельные корпускулы. Представить это в принципе невозможно, но на бумаге работает и предсказывает неплохо.
206 457573
>>57572
От самих частиц что-то зависит? Например, альфа-излучение и гамма-излучение? Или вообще если пучок молекул каких-то разогнать они будут интерферировать и все что там волны делают?
207 457574
>>57573
альфа излучение уж достаточно большие, что бы ввести себя по максимуму именно как частица и по минимуму как волна. Поэтому они уже не интерферируют.
Гамма-излучение это обычные фотоны уже и там разницы нет с обычным светом, кроме частоты волны и энергии.
Молекулы же нет, достаточно большие, что бы квантовые эффекты на них не влияли. По крайней мере в таком виде. Можно сказать, что размер их волны уже просто меньше их размера. Хотя это может и кривая фраза, но в этой квантовой механике всё и так криво и аналогии из нашей жизни привести нельзя.
1445983000171934424.jpg19 Кб, 254x226
208 457575
Почему терминатор из жидкого металла и Джон Коннор из nanomachines son не теряли память после простреливания головы? Тем самым ведь разрушались нейронные связи, на которых записана их личность. Мозг собственно им может быть и не нужен, чтобы восстановить мозг, но одна physical trauma головы и они должны просто остановиться, так как не помнят даже то, кем являются.
209 457576
>>56196 (OP)
Можно ли уничтожить черную дыру антиматерией, и поглотит ли она взрыв при точном количестве антиматерии? Что произойдет при превышении/недостаче?
Да, знаю, антиматерии физически не нашли, но я про теорию.
210 457577
>>57575
Ещё спроси каким образом свет движется быстрее скорости света.
211 457578
>>57577

>Ещё спроси каким образом любовь движется быстрее скорости света.


быстрофикс
212 457579
>>57571
Частицы и волны это всего лишь умозрительные аналогии.
Частицы это объект у которого мы отбросили все внутренние параметры, волны это объект у которых мы изучаем только внутренние параметры. Но если мы попытаемся применить статические законы написанные для частиц на волнах - выходит хуита. Когда начали пилить статистическую методы для волн, так родили квантовую теорию.
Материя же все таки строго состоит из непрерывных полей, только из-за порционности в приближении ее можно считать частицами.

Корпускулярный волновой дуализм получается своего рода упрощением для нефизиков, или всяких тупых, которые думают, что они физики, но не могут даже в линейную алгебру.
213 457580
>>57576
Нельзя. Это как попытаться тушить солнце водой. И это ещё предполагая, что в ЧД находится обычная материя, которая будет взаимодействовать с антиматерией в ней. Но что бы там не происходило, как не бахало постоянно, какие аннигиляции не происходили бы - ничего из этого горизонт событий покинуть не может. Горизонт событий это и не какой то барьер, который можно пробить, это сильно искривлённое пространство из которого не одна прямая не выходит. И энергия излучения роли не играет, оно будет лететь ровно по прямой, которая просто выхода из чд не имеет.
214 457581
>>57575

>терминатор из жидкого металла


>Тем самым ведь разрушались нейронные связи, на которых записана их личность


Бэкап в других частях тела.
В книге "Бостонский зомби: Д. Деннет и его теория сознания", например, приводится мысленный эксперимент - что, мол, два абсолютно синхронно работающих мозга будут по-прежнему образовывать одну личность, а даже если произойдёт рассинхрон, то он незаметно для личности будет устранён.
215 457582
>>57574

>Поэтому они уже не интерферируют.


Кхе-Кхе... проводили эксперимент с двумя щелями на фуллерене.
216 457583
>>57582
Ну фуллерен это не альфа частица. Квантовая механика во всю может проявляться и у молекул, но в определённых условиях лишь. Фуллерон тут исключение.
217 457584
>>57583
У одних работает у других не работает. Да, именно так физические теории и работают.
218 457585
>>57584
Атомы это уже не квантовые частицы так то. Они состоят из кварков. У протонов и нейтронов их масса покоя и с бозоном хигса то никак не связанна. Поэтому в их случае они уже куда больше частицы, чем волны даже в теории поля. Есть и размер определённый. А почему изредка и они проявляют квантовые эффекты это теории вполне себе объясняют.
Стикер63 Кб, 200x200
219 457587
>>57585

>Атомы это уже не квантовые частицы так то


Зачем ты так? Чем тебе мешает сказать что всё это квантовые частицы?)
220 457588
>>57587
И квантами какого поля являются протоны?
221 457589
>>57406

>Пиздец дегенератина. Мозг не работает по двоичной логике, ака процессор


Ты скозал?
222 457590
>>57370
А я повторю свой вопрос, в каком научпоп видосе я мог слышать фразу что коллайдер исчерпал себя, и для перехода на следующий уровень исследований нужны энергии(а соответственно и размеры коллайдера) недостижимые на земле?
И нахера тогда думают строить новый?
223 457593
Дваченые, у меня к вам вопрос.
Какой тип солнечных панелей на январь 2019 имеет самый высокий КПД?
На цену пофиг.
224 457596
>>57354
Полностью ахинея.
225 457597
>>57590
Если собираются строить новый побольше, значит, в этом есть смысл, и LHC себя не исчерпал.
Научжопу не стоит всегда безоговорочно верить.
226 457598
>>57597

> LHC себя не исчерпал.


Сука. То есть он исчерпал себя, но не исчерпал потенциал для исследований, скажем так.
227 457600
>>57588
Мы о КМ говорим а не о теории поля. Но даже так, ничто не мешает выцепить из волновой функции кусок свободного движения как целого.
229 457602
>>57597

>Научжопу не стоит всегда безоговорочно верить.


Там вроде что-то серьёзное было...
230 457603
>>57575
Сейчас, за завтраком с блинами и сгущенкой, до меня дошло: они не теряют память от того, что наномашинки запоминают присущее им расположение в теле и роль выполняемую. То бишь они возвращались на место и формировали тот же самый мозг, что и был до повреждения. Это и объясняет, отчего жидкий так охуевал без половины головы.
232 457630
>>57625
Нее, это видео я нашёл когда пытался найти те которые я видел.
Или тут его кидали же, не помню, но это точно не оно.
233 457631
>>57324
Ну ёлы-палы, ну не настолько же бесконечна бесконечность
234 457632
Так падажжи йобана, как скорость света при движении через среду может так мало замедляться?
Если всё замедление это "замедление" за счёт прохождения долгого ломаного пути от "столкновений" и "отскока" в произвольном направлении, а не прямой, то если бы столкновений было много, то эта длина была бы сильно больше пути по прямой и "замедление" было бы куда сильнее.
Но взять вот воздух, в нём скорость всего на проценты или доли процента меньше.
Следовательно много столкновений быть не может.
НО!
Если столкновений было мало, то он долго/далеко летел между столкновениями, и опять увеличение длины пути получается большим.
235 457633
>>57425
Воу, круто пояснил
236 457641
Какие области физики сейчас актуальны? Правда ли, что деньги выделяются преимущественно на исследования элементарных частиц и строительство коллайдеров?
237 457649
>>57641
БОЛЬШОЙ ОГРОМНЫЙ КОЛЛАЙДЕР
238 457650
>>57649
он самый
239 457652
Дополню вопрос выше:
Насколько выгодно сейчас идти в науку, а именно в область элементарных частиц?
Если я перекачусь за границу, нормально буду зарабатывать?
В интернете особо не нашел инфы.
241 457657
Почему гаусс-пушка которую анально форсили в играх на самом деле нерабочая хуйня без смысла?
242 457660
Есть какая-то вероятность, что в обозримом будущем эти херни начнут летать ощутимо дольше? Раз в 5-10 хотябы.
Чёт пока набирал подумалось что нет, т.к. если бы это на моей жизни случилось, то уже бы во всю везде об этом трубули что это возможно и работы в этом направлении активно идут/имеются работающие прототипы и планировались к постройке заводы
243 457663
>>57660

> Раз в 5-10 хотябы.


Маловероятно. В определённый момент вся система озалупиться. Суть в том, что для более продолжительного полёта требуется более мощные батареи, которые больше, а значит больший вес и нужен более мощный двигатель, который сильнее жрёт батареи. Вот и мы и озалупились попав в бесконечный цикл увеличения размеров вплоть до вертолёта, где уже проще использовать дефолтные топливные двигатели.
На данный момент эргономика конструкции этих хуёвин не позволяет значительно увеличивать вес и поэтому нужны более совершенные источники питания. А у них прогресс довольно медленный.
244 457664
>>57663
Очевидно что вопрос был на счёт прорывов в аккумуляторах.
Ну по крайней мере мне так казалось.
245 457665
>>57664
Вопрос был будут ли эти хуйни летать в 5-10 раз дольше.
Я ответил - маловероятно. Предел эргономичности уже достигнут, а прорывов в аккумуляторах на горизонте нет.
Без упоминания конструкции ответ был бы не полным потому, что в некоторых случаях оптимизация творит чудеса.
твиттер-интернет-Илон-Маск-авто-3125910.webm104 Кб, webm,
480x266, 0:04
246 457680
В тред призывается оптик-кун. Поясни пожалуйста за коэффициент диффузии света в веществе (стекло), который измеряется в см-1. Нигде нет информации, и никаких сборников с этим значением.
247 457682
Люблю на досуге поразмышлять о таких вещах как интерпретации КМ, искривление пространства-времени, четвертое измерение и тд. Просто доставляет удовольствие упражнять воображение .

ВНЕЗАПНО осознал что всем этим концептам более 100 лет! Пиздос, где свежак!? Какие мозговыносящие концепты последних десятилетий можете назвать? Чтобы осознать и прихуеть
248 457684
>>57682

>Чтобы осознать и прихуеть


Всем этим концептам уже овер 4к лет, просто они существовали без привязки к конкретным физическим сущностям.
Концепты, которые мы нарабатываем сейчас, войдут в жизнь через столько же времени.
Прихуел?
249 457685
>>57682

> Какие мозговыносящие концепты последних десятилетий можете назвать? Чтобы осознать и прихуеть


Тачскрин интерфейсы, много деталей и тонкостей в них. Браузеростроение и АПИ расширений
250 457686
>>57685
Например, в итоге подебил АПИ WebExtensions, а не гораздо более гибкая система аддонов старого Фирефоха, неожиданно. Потому что АПИ независим от смены кишков бровзера
251 457687
>>57685
Еще пример: только Аппле догадалась показывать интерфейс бровзера на тачскрине при касании рядом с краем экрана
252 457689
>>57684

>Концепты, которые мы нарабатываем сейчас, войдут в жизнь через столько же времени.



Так какие?
253 457692
>>57685

> Браузеростроение и АПИ расширений


Залупа говна.
Последнее, что пошло в массовость это жидкие кристаллы, которые по сути переоткрыли в 60х годах.
254 457694
>>57684

>Всем этим концептам уже овер 4к лет



Антиинтуитивной логике квантового мира овер 4к лет?
Да такой херни ни в одной древней культуре не было
255 457695
>>57687
Реквест был на теории о пространстве, времени и материи, а ты мне про какие-то веб-стандарты втираешь
256 457697
>>57694
Апория Зенона, вполне себе из КМ. Даже есть квантовый эффект Зенона.
257 457700
>>57697
Зенон парадокс только поставил, а не теоретизировал как его разрешить
258 457702
>>57700
Ну понятное дело, но "неизмеримость точного положения в движении", это сильно, особенно для овер 2к лет назад.
259 457703
>>57660
Вероятность есть всегда. Но очень маловероятно. Ибо тут нужен качественный революционный скачок в аккумуляторах. А его не предвидется.
260 457704
БЕРЕМ стирлинг, на 1квч, охлаждаем головку до -30-40 фрионом с помощью 300вт - пользуемся 700втч на халяву

где подъеб?
261 457709
>>57704
нагревать чем будешь?
262 457712
Это чистая случайность, что возраст вселенной более-менее представим и соизмерим с обычными историческими/геологическими периодами (и то же относительно размеров, хотя они уже поболее)? Ну т.е. почему именно миллиарды лет, а не 10^10^10... и так миллион раз, например? Или просто наука до таких масштабов ещё не добралась?
263 457713
Хотел узнать из чего "состоит" магнитное поле в итоге на ответах меил.ру прочитал про виртуальные фотоны.
Погуглил, в умных книжках про это что-то есть (пикрил), но мне вряд ли понять.
Объясните, что это такое?
264 457714
>>57713
Почему в твоем вопросе именно магнитное поле? С электрическим тебе все понятно?
265 457715
>>57680
По-размерности скорее всего ты про attenuation coefficient. Погугли и репорть что нашел
266 457716
>>57714
Да просто к экзамену по геологии читал про МП Земли
267 457719
>>57709
Он же работает от любой разницы температур.
268 457720
>>57719

>БЕРЕМ стирлинг, на 1квч


>Он же работает от любой разницы температур.


Даже в 1С ? И какая по-твоему разница температур должна быть, чтобы снять в итоге 1кВт/ч ? Это дохрена для такого говна.
269 457721
>>57716
Скажи на экзамене, что магнитное поле это виртуальные фотоны, бегающие по силовым линиям.
270 457722
Существуют сотни гипотез, объясняющих существование шаровой молнии, но практически никто не занимается их проверкой на практике. Почему?
271 457724
>>57722
Сложно воспроизвести то чего нет.
272 457725
>>57722
Потому что все или почти все во времена твоего дедушки опровергли. сука, как же я её боялся в детстве во время гроз, реально думал, что вылезет из розетки лайтингболл и будет с визгами за мной гоняться, щипая разрядами за жопу
273 457729
>>57712
В смысле случайность?
Это случайно, что возраст твоей кожи сопоставим с твоим биологическим возрастом?
274 457731
>>57713
Если бы учёные сами толком понимали, что там происходит, то они бы и не придумывали такие термины. Это в первую очередь просто математическая модель. То насколько они реальны же, вопрос весьма спорный, ибо хоть в некоторых моментах они и показывают своё существование, но в целом их реально существование противоречит законам физики. Поэтому тупо математически их реальными объявить и нельзя. Так что речь скорее идёт просто ещё о неописанном свойстве квантового поля.
275 457732
>>57720

>И какая по-твоему разница температур должна быть, чтобы снять в итоге 1кВт/ч ?


Зависит от размеров
Я другой анон, не задавший первичный вопрос.
Сначала он показался логичным, типа же с одной стороны ты температуру прикладываешь, а с другой она сама постоянно обновляется, ветерком например. И типа вклад в энергию работы идут с одной и второй стороны, а со второй само обновление происходит, ветерком например.
Но лол, если бы ты получал больше чем затратил, то выходит что ты и получал 1квт, вместо 300вт приложеных, но ещё бы и нагревал охлаждающую среду, которая бы постоянно обновоялась.
А нагрев это расходывание энергии.
276 457733
>>57713

>Хотел узнать из чего "состоит" магнитное поле


Зачем узнавать из чего оно состоит, поле оно ж и есть поле.
277 457734
>>57712

>Это чистая случайность, что возраст вселенной более-менее представим и соизмерим с обычными историческими/геологическими периодами (и то же относительно размеров, хотя они уже поболее)? Ну т.е. почему именно миллиарды лет, а не 10^10^10... и так миллион раз, например? Или просто наука до таких масштабов ещё не добралась?


Тут наверное ошибка выжевшего.
Если бы возраст был больше, то люди бы вообще не узнали о вселенной, потому что бы всё разлетелось, или вообще уже ничего не происходило и ничего не существовало, потому что всё разлетелось.
Если бы оно разлеталось медленнее, так что через 10^10^10... всё бы не разлетелось, то оно бы разлеталось слишком слабо и было стянуто обратно гравитацией в сверхплотную свергорячую структуру, где тоже ты бы не появился никогда.

Просто наблюдатель появился тогда, когда для этого были благоприятные условия, что логично.
278 457746
Далек от физики, но есть вопросы.
Вопросы насчет статического электричества. Есть ли у него плюсы?
Возможно ли накапливать заряд?
Если да, то реализуемо строительство запасных генераторов в человеческих колониях на Луне или Марсе?
279 457747
>>57746

>Далек от физики, но есть вопросы.


>Возможно ли накапливать заряд?


В школе двоечником был?
280 457748
>>57747
3 по физике было
281 457754
>>57709

>нагревать чем будешь?



Окружающей средой. Можно в землю вкапать.
282 457755
Если поместить утюг в экзосферу, где температуры порядка сотен градусов Цельсия, он нагреется до теспературы окружающей среды со временем?
283 457757
>>57755
А почему ты думаешь, что он не будет нагреваться?
284 457760
>>57757
Он же выключен
285 457761
>>57746
Да, можно, потри эбонитовой палочкой о шерсть - вот тебе и разряд.
Но другие виды аккумуляторов намного эффективнее
286 457762
>>57757
Ну хуй знает. Утюг по идее будет терять энергию через излучение с мощностью, зависящей только от температуры, и если делать среду вокруг него все более разреженной (при сохранении постоянной температуры), то мощность потерь через излучение станет больше мощности обогрева утюга от окружающей среды (корорая наверн пропорциональна концентрации частиц вокруг).
287 457763
>>57761
я имел в виду снимать статическое электричество с генератора и им заряжать аккумуляторы обычные.
288 457764
>>57712
Определи что значит "представим и соизмерим". Ты просто к конкретно этим цифрам привык, был бы возраст вселенной 10^100 лет - ты бы и к ним привык.

И вообще ты уверен что этот период действительно представим? 13 миллиардов лет это 3.998592e+17 секунд
289 457765
>>57712
>>57764
Я к тому что единицы измерения так подобраны что-бы было удобно оперировать числами. Если измерять возраст вселенной в секундах или наносекундах то там заоблачные числа получатся как ты и хотел
290 457766
Хорошо,смотрите.
Допустим, у нас есть помещение с абсолютно одинаковыми по размерам стенами, на каждой закреплен огромный электромагнит.
В середине металлическая конструкция способная плавно вращаться по окружности вырабатывая при этом электричество.
Если попеременно на миллисекунды подавать последовательно питание на каждый магнит, то эта металлическая хреновина придет в движение, вырабатывая ток.
Внимание вопрос, сможет ли покрыть вырабатываемая электроэнергия ту что расходуется на электромагниты?
291 457767
>>57766
учебник физики за 8 класс
закон сохранения энергии
нет
292 457772
>>57766

>Внимание вопрос, сможет ли покрыть вырабатываемая электроэнергия ту что расходуется на электромагниты?


Зачем вообще начинать рассуждать об этом, когда у тебя есть закон сохранения?
293 457774
>>57772
если б Ньютон на двачах сидел тоже бы в дураках ходил, беру на заметку, придется читать ваш учебник по физике, без него никуда
294 457775
>>57632
Матрица делает вид что не замечает вопросов о проёбах в своём устройстве, понятно...
295 457780
>>57775
просто ты пытаешься вместо науки впихнуть свои ощущения. А ты докажи сначала, проведи опыт
296 457782
>>57775
Всё это поглощение с последующим излучением происходит мгновенно практически. Отсюда и замедление такое маленькое. Или ты думаешь, что электрон впитает квант света, подумает пол секунды и только после его испустит? Ты бы тогда своё отражение в зеркале видел бы с большой задержкой.
297 457783
Друзья, мне кажется человечество неправильно поняло природу света. Вернее способы его использования. (Помимо того, что имеем сейчас)
298 457784
>>57772

>Зачем вообще начинать рассуждать об этом, когда у тебя есть закон сохранения?


Вопрос от другого дивана:
Если в кококвантовой физике закон сохранения энергии может нарушаться (Ну в научжопе так пишут по крайней мере, нулевая энергия, вертуальные честицы, вся хуйня), почему этим нельзя воспользоваться, чтобы построить вечный двигатель I рода?
299 457785
Можно ли в теории, при помощи системы квантовозапутанных фотонов (или других частиц) побайтово передавать данные из точки А в точку Б?
300 457787
>>57784

>Если в кококвантовой физике закон сохранения энергии может нарушаться


Ну там нарушение чисто понятийное (измерение ууу... энергию точно не узнать), фактически ничего не нарушается.
301 457788
>>57762

>Утюг по идее будет терять энергию через излучение с мощностью, зависящей только от температуры


Откуда ты это взял?
мимокрок
302 457790
>>57788
Ну в смысле для данного утюга мощность теплового излучения только от температуры будет зависеть. А от чего еще?
303 457793
>>57783

>мне кажется человечество неправильно поняло пр


Аргументы?
304 457795
>>57793
один и единственный аргумент - почему мы используем его в быту только в виде лампочек. Если есть столько применений, видимому свету, солнечному свету
305 457796
>>57795
Как еще использовать?
306 457797
307 457798
>>57796
да даже нет элементарных приблуд для фокусировки солнечного света весной летом, для с/х
308 457799
>>57798

>нет элементарных приблуд для фокусировки солнечного света весной летом, для с/х


Эм... зачем? И как это соотносится с неправильно поняло природу света?
309 457800
>>57799
посмотри как фотончики используют в НОУКЕ
и как их используют обыватели. по моему все очевидно
310 457801
>>57725
Двачую
311 457802
>>57800

>очевидно


Ну поделись этой очевидностью.
312 457803
>>57802
можно я завтра отвечу кратко и по существу? Не хочу флейм разводить, будет время подумать
313 457806
>>57780

>просто ты пытаешься вместо науки впихнуть свои ощущения. А ты докажи сначала, проведи опыт


Какие ощущения, поехавший? Ты отрицаешь приведённые всюду значения скорости света в среде и в вакууме? Или отрицаешь способ распространения света в среде?
314 457807
>>57782
Ты неправильно понял вопрос, перечитай ещё раз, в нём вообще не учитывается время переизлучения, нулю равным можно сказать принимается в контексте вопроса.
315 457808
>>57807
Просто придется объяснять тогда, что фотон частично сталкивается, и частично проходит через молекулу (как ебаный кот Шредингера), при этом эти две части, потом еще и интерферировать могут.
316 457809
>>57808
извини, но ты дурак.
317 457810
>>57808
Переизлучение происходит в случайном направлении (?)
Половина фотона переизлучается в случайном направлении, а половина проходит насквозь?
318 457811
>>57810

>Переизлучение происходит в случайном направлении (?)


Нет, там распределения свои.

>Половина фотона переизлучается в случайном направлении, а половина проходит насквозь?


Какая конкретно часть прямо проходит, а какая размазывается по углам надо смотреть на конкретный случай. Это к спектроскопистам вопрос. Просто не надо воспринимать фотон как шарик, или как частицу с конкретной траекторией.
>>57809
Да, я такой.
319 457812
1. С какой максимальной скоростью двигался человек относительно наблюдателя, стоящего неподвижно на поверхности Земли?
2. Обладает ли 1 атом например ферума такими же физическими и химическими свойствами, как килограмм ферума?
3. Если обладает, то где хранится информация об его свойствах, если в стандартной модели атома есть только электроны, протоны, нейтроны и условные электронные радиусы? Все в каждом химическом элементе одинаковые, а радиусы на самом деле не вещество.
320 457813
>>57812

>Обладает ли 1 атом например ферума такими же физическими и химическими свойствами, как килограмм ферума?


Нет. Про химические свойства одного атома практически не говорят.

>где хранится информация об его свойствах, если в стандартной модели атома есть только электроны, протоны, нейтроны и условные электронные радиусы?


Там даже радиусы не нужны. Если у меня бы например была машина Тьюринга с бесконечной вычислительной мощностью, я бы смог обсчитать свойства килограмма железа на компе. Вся химическая проблематика в квантовой механике содержится.
321 457814
>>57812
ты дурачек? Килограмм железа не равно одному атому
322 457815
>>57813
Лол что значит нет? А сколько тогда нужно атомов, чтобы их химические свойства начали соответствовать 1 килограмму элемента?

>>57814
А ты вообще иди нахуй. Можешь ещё для профилактики своей деменции перечитать чётко поставленный вопрос в треде вопросов, может с 50 раза до тебя дойдет о чем речь.
323 457816
>>57811
Как вывести именно такую скорость(в какой-нибудь среде, для упрощения чистый одноэлементный газ) через "микровзаимодействия"?
324 457817
>>57815
Да это ты нахуй иди поехавший.
Какую же хуйню ты спорол. физические блядь свойства атома будут ли равны куску железа. Ебанько
325 457818
>>57817

>Физические свойства - свойства присущие веществу вне химического взаимодействия: температура плавления, температура кипения, плотность, агрегатное состояние, цвет, запах, растворимость, электропроводность, теплопроводность.


Нассал тебе тугой струёй прямо в твоей тупорылый ебальник, поражённый деменцией.
326 457820
>>57812

>Обладает ли 1 атом например ферума такими же физическими и химическими свойствами, как килограмм ферума?


Какими физическими? Удельный вес? Цвет? Теплопроводность? Плавкость? У одного атома ничего этого нет, физ. свойства металла будут определяться его структурой, составленной из множества атомов.

>где хранится информация об его свойствах


состав ядра атома и конфигурация электронных оболочек.
327 457821
>>57820

>У одного атома ничего этого нет, физ. свойства металла будут определяться его структурой, составленной из множества атомов.



>А сколько тогда нужно атомов, чтобы их химические и физические свойства начали соответствовать 1 килограмму элемента?

328 457822
>>57821

>чтобы их химические


один атом уже имеет такие хим. св-ва, например, если это одноатомный газ.

>физические свойства


Без понятия, но, скорее всего, много.
image.png35 Кб, 220x285
329 457824
>>57818

>температура плавления атома

330 457825
>>57807
Фотонов может быть множество во одном состоянии. Ещё не все будут взаимодействовать. Это значит, что в одном состоянии в одной точке может быть десятки фотонов. Какие то из них пролетят без сопротивления, какие то слегка затормозятся, это и может привести к помехам в сигналах. Вообще когда ты испукаешь радио сигнал, ты как бы испускаешь всего-лишь один фотон, который равномерно летит во все точки от твоей антенны. То есть в таком виде фотон ведёт себя как обычная волна.

Но это из-за того, что он имеет нулевой спин. Нет ограничений в том сколько фотонов может находится в одной точке в одном состоянии, когда электронов может быть лишь два. Поэтому фотоны и так хороши для передачи информации. Летит миллиарды фотонов с одной и той же информацией, даже если часть с чем то взаимодействует, например, с твоим телефоном, миллиарды других полетят дальше и передадут сигнал другим телефонам ровно так же.

Но их не бесконечное колличество, поэтому по мере расстоянии и в зависимости от среды часть информация теряется, всё больше помех в ней.

Ты неправильно просто фотоны воспринимаешь, как какие то шарики.
331 457826
>>57815

>сколько тогда нужно атомов, чтобы их химические свойства начали соответствовать 1 килограмму элемента?


Достаточно много, смотря что ты считаешь "соответствовать", может тебе точность в 10^-8 подойдет, а может нет. Чем больше ты атомов будет добавлять, тем больше свойства будут стремиться к свойствам бесконечного куска железа. Понятно, что уже на 10^14 атомов свойства станут близки, точную границу сейчас не назову, надо нарыть в учебниках. У наночастиц свойства например отличаются от обычных веществ, так как существенная часть атомов на границе частицы, а атомы на границе активнее. Например SiO2 инертная хрень, её можно жрать и с тобой ничего не будет, но пыль вызывает рак. Как так? А у мелких частиц большая восстановительная способность, на поверхности частиц образуются пероксидные мостики.
332 457827
>>57825

>Вообще когда ты испукаешь радио сигнал, ты как бы испускаешь всего-лишь один фотон, который равномерно летит во все точки от твоей антенны.


Таки сферически-симметричная поперечная (а ЭМИ - поперечные волны) волна зопрещена математически, в Сивухине где-то такое было.
мимо дропнул физфак
333 457828
>>57825
Спин у фотона точнее равен 1.

>>57827
Ну я уж не настолько эксперт, так, любитель. Может не симметричная, но при таком излучении фотон ведёт себя как обыкновенная волна, как частица ведёт себя же при взаимодействии.
334 457829
>>57822
Одноатомный газ, это не один атом. Одноэтажный поселок, это не одноэтажный дом.
335 457838
>>57828

>любитель


Любитель который совсем не вдуплил вопрос, и накатал такую простыню в пустую.
Где там хоть слово было о помехах?
336 457839
>>57821

>А сколько тогда нужно атомов


Порядка числа Авогадро. В действительности статистические закономерности начинают проявлять уже с 1000 частиц.
337 457847
>>57829
И как по-твоему в межзвездном пространстве образуются молекулы, в том числе сложные, если там плотность почти что ничего на кубометр?
338 457849
Что происходит на видео? Насыпал немного мелкого сухого песка в таз, заметил, что он как-то неспокойно пересыпается по нему. Через несколько минут начал прилипать к стенкам таза, а потом я заметил, что песок следует за движениями моих пальцев по внешней стороне таза. При чём иногда он магнитится, а иногда наоборот отталкивается. В одних частях таза эффект наблюдается в разы сильнее, чем в других. Я понимаю, что дело в статическом электричестве, но почему оно там возникает? Почему оно так влияет на песок? Можно ли как-то усилить эффект для большей показательности?
339 457850
>>57812

>где хранится информация об его свойствах


В структуре ядра все хранится, ядро же порождает электронные оболочки, которые добавляют несколько своих свойств.
340 457851
>>57849

>но почему оно там возникает?


песок трется об пластмассу, например при пескоструйной обработке электризоваться может вообще дико.
341 457861
Что будет, если сожрать тарелку вирусов спида/гриппа/бешенства? Каковы они на вкус и цвет?
342 457865
>>57847
Местами он уплотняется, и как бы когда мы говорим о "разряженом газе", или вообще о любых веществах, мы всегда говорим о большом количестве атомов. Химические свойства это про свойства веществ.
343 457872
>>57785
Нет
344 457874
Произвёл какой-то человек товар на 5$.
Денежной системе нужно напечатать количество денег равное произведенному товару?
345 457878
>>57874

>Произвёл какой-то человек товар на 5$


Как человек узнал что товар на 5$?
346 457879
>>57878
Продал его за столько/его за столько готовы купить
chrome2019-01-2317-16-44.png93 Кб, 863x453
347 457891
Если земное ядро взять в клешни и тянуть тросом со скоростью 10кмч, нам пизда-пизда будет или терпимо? В смысле, вся остальная Земля начнёт двигаться вместе с ним весьма равномерно или начнётся дикое перемешивание и смещение ядра?
348 457898
>>57891
Не знаю.
349 457899
Почему электромобили называют экологически чистым транспортом? Количество энергии, необходимое для передвижения ведь тоже нужно произвести, а это полюбому какие-то выбросы, если это не атомная электростанция. Или все же выбросов меньше? Можно ли населению небольшого города на 100000к жителей моментально пересесть на электромобили, на сколько нужно модернизировать электросетевую инфраструктуру для этого?
Кто-нибудь занимался такими расчетами?
350 457901
>>57899

>Почему электромобили называют экологически чистым транспортом?


Прост так хотят. Они ебнутые.

>Или все же выбросов меньше?


Питают от ТЭС, так что не сильно меньше. Даже больше, производство батарей очень сильно загрязняет окружающую среду (в Китае). Поэтому типа выгодней загрязнять не свою страну а Китай.
351 457904
>>57899
Передел рынка. Вывод кучи старых дешёвых автомобилей с рынка. Но это надо как то обосновать.
352 457909
>>57898
ну ты подумой(((9
353 457913
>>57899
КПД ТЭЦ или даже угольных ГРЭС выше чем у ДВС, а выбросов на кг топлива меньше, кроме того можно запилить системы очистки прямо на станции и пердеть только водой и углекислотой.
ДВС в городе выделяет и сажу, и окислы азота и серы, и недогоревшие циклические соединения. Всё это не очень полезно для здоровья, потому лучше выбросы вывести за пределы города.
С другой стороны, сами батареи, конечно, не экологичны, да и заправка там жопная.
Когда будут сделаны батареи с быстрым зарядом (5 минут, с охлаждением блока батарей, например), то пересаживаться на электро уже будут массово, даже если они и дороже.
354 457919
>>57913

>КПД ТЭЦ или даже угольных ГРЭС выше чем у ДВС


Какое КПД доставки энергии от сжигания угля до автомобиля?
355 457921
>>57812
1. Думается мне, что макс. Скорость косм. Корабля -это примерно оно и есть. ~40000 км/ч
356 457922
>>57861
Лол
357 457923
>>57861
Ты заболеешь
358 457943
были ли жиробасы в доисторические времена?
359 457944
>>57943
самка в период беременности частично да, могла быть
360 457946
Хочу провести эксперимент. Реально ли в домашних условиях создать источник гамма излучения?
361 457950
>>57891
Оно ж жидкое
Вроде
Нужно узнавать по уравнению состояния вещества
Девочки.jpg99 Кб, 660x497
ня 362 457952
Интерпретируйте каждую спину
Вторая знаю "Уравнения Максвелла в дифференциальной форме"
А остальные плиз помогите
еееуеупе.jpg128 Кб, 1267x599
363 457953
https://www.youtube.com/channel/UCDz7dYKC6Mvj7JKfzQGEm7w лучший российский научпоп
364 457955
>>57952
норм шпоры. типа делаешь вид, что ебёшь, а на самом деле экзамен сдаёшь
365 457957
>>57950
ВРоде твёрдое, но не суть, мы его герметично обернём
366 457958
>>57957
Конечно пиздец будет, инерция остальных вращающихся слоёв сохранится и они будут набегать на уже "выступающее" ядро, как вода в ручье набегает на выступающий камень.
123эффект.png5 Кб, 331x75
367 457959
Сап наукач,надо расчитать вероятность туннелирования ядра дейтерия и трития,что вставить вместо m?
368 457962
>>57952
По этой картинке можно определять образованность анонов - кто больше угадает. Разрешение хуёвое, конечно.

1.Лоренцев буст по оси икс; дисперсионное соотношение энергии-импульса; уравнения Эйнштейна; выражение для символов Кристоффеля через метрический тензор; уравнение геодезических.
2. Уравнения Максвелла для вакуума в системе СИ.
3. 2-ой закон Ньютона; уравнение стационарности действия в 4ёхмерном пространстве; теорема Нётер для тензора энергии импульса в декартовых координатах в отсутствие гравитации; волновое уравнение для волн, перемещающихся со скоростью света в трёхмерном пространстве; уравнение распределения частиц по энергии в системе из невзаимодействующих частиц, подчиняющихся распределению Максвелла-Больцмана.
4. Канонические коммутационные соотношения; уравнение Шрёдингера в координатной записи для нерелятивистской частицы в поле; диаграмма Фейнмана для бета-минус распада нейтрона; группа симметрии калибровочной теории Стандартной модели.

Изи, следующий.

Выбор уравнений однозначно говорит о том, что те, кто писал их на спинах, не понимают смысла уравнений и их место в теории. Уравнения Максвелла выбиваются из других уравнений, т.к. практически всё остальное записано тензорно, что показывает, что те, кто писали это, не понимают, что это за уравнения, т.к. иначе записали бы их через дифформы и оператор Ходжа.
369 457968
по какой формуле можно рассчитать кпд мартышкинного труда?
370 457969
>>57968
Это относительная величина.

А хотя нет, лол, можно в абсолютных величинах посчитать.
371 457970
Предлагаю всем /sci объединится и совместными усилиями придумать новый вид реактора.
372 457971
>>57970
На говне?
373 457972
>>57971
+1 заметка в копилку - Реактор на биотопливе.
374 457973
>>57962
Забыл уравнение Дирака для свободной частицы между диаграммой Фейнмана и группой симметрии. В автобусе неудобно набирать, все норовят поглазеть на голые спины как только я переключаюсь на фото.
375 457974
Обогнал меня анон >>57962. Правда, я не всё узнал, в теории поля я профан.
Интереснее то, что я нашёл вбсолютно такой же тред на dxdy, где наш анон >>57952 выдаёт себя за девушку, чтобы, видимо, побыстрей ответили. https://dxdy.ru/topic132648.html
Забавно, что анон из /sci/ дал боеле обстоятельный ответ.
Вопрос к >>57952: что за задание? Откуда, куда? В универе такое, или нужно покрасоваться перед знакомыми?
376 457975
>>57962

> иначе записали бы их через дифформы и оператор Ходжа.


Ненужное в 90% случаев пижонство.
377 457982
>>57975
В теорфизике намного удобней обращаться с уравнениями Максвелла в тензорной записи или через дифференциальные формы. Всё, что касается Янга-Миллза, узлов, связностей. Тем более, вся картинка - пижонство, по такой логике чем непонятней, тем круче.
378 457989
>>57715

Attenuation coefficient не то, там именно что коэффициент диффузии. Как я понял в диффузной оптике фотон видимого спектра рассматривают как частицу, которая может диффундировать в веществе. То есть они что-то типа фотонный газ рассматривают. Но нигде никаких справочников по этому нету. Только презентации, в которых что-то не очень серьезное пишут.
379 457990
>>57959

Массу дейтерия, x1 и x2 ширина барьера.
380 457991
>>57962

Тензорно только первая и последняя спина, вторая и четвертая обошлись без этого. Запись уравнений Максвелла в тензорной форме сразу детектирует теоретика, который не понимает зачем они нужны. Любой оптик именно так и запишет, потому что именно так они и применяются для решения в различных световодах, например.
381 458000
Сколько энергии сможет вырабатывать микроустановка на обычном водопроводе?
382 458006
>>57991
Я же сказал, что в теорфизике. Reading comprehension fail у тебя.
383 458010
Насколько разрушительно будет получасовое 10бальное землетрясение на всей Земле?
384 458011
>>58010
скажу так, повезет тем кто живет в селах и деревнях
385 458012
>>58010
примерно как постоянная получасовая бомбардировка земли царь-бомбами. или даже хуже.
386 458015
Посоветуйте что-то (чтиво/лекции/прочее) на тему мораль с точки зрения эволюции и нейронаук.
387 458017
Сап, посоветуйте хороший учебник по физике. Я знаю только про Фейнмановские лекции по физике и Ландау Лифшица. Как я понял Ландау Лифшиц не мой уровень и не знаю зайдет ли Фейнман. Сам я заканчиваю 11 класс. Также реквестую учебники по математике чтобы эту физику понимать. Заранее спасибо.
388 458020
>>58017
На физфак собираешься? Возьми стандартный курс сивухина. По математике, либо Зорич, либо Фихтенгольц.
389 458024
>>58000

>Сколько энергии сможет вырабатывать микроустановка на обычном водопроводе?


А самому прикинуть не судьба? Предположи, что чудо-установка преобразует в электричество (или что там тебе надо) всю кинетическую энергию струи из-под крана. Скорость струи среднюю, по оси скорость воды будет больше чем по краям вывести можно из расхода воды за минуту и диаметра дырки в кране.
image.png197 Кб, 600x449
390 458025
Матемач, выручай!
Есть ряд ежедневно меняющихся значений, которые нужно накопительным итогом помесячно просуммировать, т.е. выглядит так:

x^30 + x^60 + x^90 + x^120 ...

Как это привести к виду удобоваримой для Экселя формулы?
Общий вид нужен для возможности добавления данных в ряд, конечно.
391 458027
>>58025

>x^30 + x^60 + x^90 + x^120 ...


Геометрическая же прогрессия, для облегчения замени x^30 на t, получится t+t^2+t^3+t^4 ...
Ну а дальше формулы для суммы геом.прогрессии
392 458044
А есть теории, которые воспринимали бы фотон и прочие частицы, которые находятся в суперпозиции, как вневременные?

Ну то есть все мы помним эти теории о том что, чтобы вернуться в прошлое, надо двигаться быстрее света, но свет ведь двигается со скоростью света. То есть он и не в настоящем и не в прошлом и не в будущем, он вне времени. А поглощаясь нашими глазами мы не то что бы схлопываем суперпозицию до единственного варианта, а мы просто этот самый единственный вариант видим, в виду нашего линейного восприятия природы времени. В таком случае наблюдатель бы не влиял, как нам сейчас кажется, на объективную реальность, он бы просто воспринимал ее как одну единственную.

Короче связывал ли кто-то природу суперпозиции со временем? Потому что теория редукции может оказаться обычным человеческий эгоцентризмом и мы не схлопываем множество возможностей до одной единственной своим присутствием, а мы просто видим одну вероятность и не видим другую, двигаясь по времени только вперед.

Или я вообще все неправильно понимаю?
393 458052
>>58027

>формулы для суммы геом.прогрессии


Спасибо!

Блин, действительно работает, как же хорошо, что есть ученые!
для четырех членов у меня вышла формула (x^30 * ((x^30)^4 - 1))/(x^30 - 1)
И для х больше и меньше единицы совпадает с расчетом вручную.
394 458053
>>58044
Да, ты нихуя не понимаешь.
В современном представление физическая реальность полностью детерминирована по всему фазовому пространству. Однако есть "огрызки" реальности (классические физические системы), которые неполно отображают кусок полной физической реальности. Из-за того что отображение не полное следует, что переход внутри огрызка в другой огрызок необратимый. Т.е время становится направленным.
Кстати, любая частица это не самостоятельная вещь, это неполноценное отображение реальности на огрызок реальности - детектор/наблюдатель/твою мамку шлюху.
395 458054
>>58020
Да, спасибо.
396 458055
>>58053

>В современном представление физическая реальность полностью детерминирована по всему фазовому пространству.


Это неверно и такое чувство, что ты не понимаешь, что такое фазовое пространство вообще.

>Кстати, любая частица это не самостоятельная вещь, это неполноценное отображение реальности на огрызок реальности - детектор/наблюдатель/твою мамку шлюху.


Это правда. Особенно с этим проблемы в квантовой оптике, приходиться достаточно чётко моделировать всякие лавинные детекторы чтобы наверняка определить однофотонный источник.
397 458060
>>57962
Вау!
А где нужно учиться и что читать, чтобы достигнуть таких же знаний и глубокого понимания каждой формулы?
398 458067
>>58017
Матвеева, Иродова, Ландсберга еще можно почитать для начала, по дифурами и матану Позняка могу еще посоветовать, и конечно Демидович задачник.
399 458068
>>58060
Тащемто тут ничего сложного нет, просто берешь и учишься на физфаке без задней мысли.
400 458080
Гайс читаю Рассела историю философии. Наткнулся на следующее - "Если вы скажете, что "Гамлет" - это имя воображаемой личности, то, строго говоря, это неверно; вы должны были бы сказать: "Воображают, что "Гамлет" - имя действительной личности"; Вообще не отдупляю в чем разница, помогите пожалуйста.
401 458096
>>58080
А зачем ты его читаешь?
402 458097
>>58096
Хочу примерно общую картину составить и потом пойти по каждому философу. Мне нужен более менее системный взгляд на происходящее в философии. Кстати параллельно гайс посоветуйте кого почитать по Гераклиту. Наткнулся на лекции Хайдегера, но могу не потянуть т.к. с логикой знаком плохо
403 458098
>>58096
И, пожалуйста, отвечайте предметно. Причины моих действий прошу оставить мне.
404 458099
>>58080
Ну типа что "воображаемая личность" это плохое словосочетание, мол такого не существует. Мол правильнее сказать, что люди воображают что Гамлет это действительная личность.
405 458100
>>58099
Все равно не понимаю. Воображаемый Гамлет есть имя воображаемой личности - воображаемое есть воображаемое. В чем проблема ?
406 458101
>>58100

>Воображаемый Гамлет есть имя воображаемой личности


Воображаемая личность это оксюморон. Типа личность не может быть воображаемой. Никто не может вообразить личность.
407 458102
>>58100

>Воображаемый Гамлет есть имя воображаемой личности


Тебе правда надо логику учить вместо Хуйдеггеров с Хераглитами. Они-ж умерли давно, а тебе еще жить. Гуманитарием.
408 458103
>>58101
Если личность может быть только действительной, тогда "действительная личность" - это тавтология... не понятно
409 458104
>>58102
Разумно, т.е. даже до греков нужно отдельно изучать логику ? Посоветуй кого читать, что учить.
410 458105
>>58103
Он пытается свести всё к реально существующим вещам. Потому что он рассказывает про Парменида. И он берет и рассматривает слово "Гамлет", которое должно что-то обозначать, но предмета нет в природе. Поэтому все утверждения с Гамлетом бессмысленны, так как непроверяемы. Но Рассел говорит, что если мы скажем: Шексир писал, что Гамлет... то это уже истинно и можно проверить.
411 458107
>>58104
Гугли учебник по формальной логике и вперед.
Задачки решать обязательно. Я проверю.
412 458137
возник вопрос очень странный, я дурак похоже.

Смотрите. Свет от солнца поглощается объектом, то что не поглощается отражается, фотон попадает нам в глаза и мы видим яблоко. Так это работает? И так с каждым материалом куда падает свет. Так скажите, почему фотоны не пересекаются друг с другом?
413 458139
>>58137
Фотоны очень даже пересекаются.
414 458140
>>58137

> то что не поглощается отражается,


.. или проходит дальше.

>Так скажите, почему фотоны не пересекаются друг с другом?


Что ты понимаешь под пересечением? Пересечение траекторий или взаимодействие? Если последнее, то сечение гамма-гамма рассеяния настолько мало, что можно считать, чот фотоны не взаимодействуют друг с другом.
415 458142
>>58137
Фотон это бозон. Он может занимать одно и то же состояние с другими фотонами
416 458148
>>58142
Но ведь информация разная, один объект светит красным, другой зеленым.
417 458149
>>57874
Бамп
Так оно должно функционировать?
Товара/услуги стало больше, а ведь деньги это эквивалент товаров/услуг. Значит и денег должно стать больше.
418 458150
>>58149
Это не так работает. Пошел ты к парикмахеру. Тебе оказали услугу на 5$. Твои деньги находятся у другого человека, на этом все. Инфляция же по другим причинам
419 458151
>>58148
Частицы не могут непосредственно сами с собой взаимодействовать (кванты скалярных полей таки могут, но в природе "чистые" скалярные поля не встречаются.). Обязательно должно быть либо фермион-бозон, либо фермион-бозон-фермион.
Из-за своей статистики бозон не может передать другому бозону свое состояние, зато они могут одновременно взаимодействовать с одним фермионом, на этом собственно основан лазер.
Впрочем в очень глубокой потенциальной яме (поле тяжелых ядер или особый дефект в решетки) фотон может рассеиваться на другом.
420 458152
>>58151
Что значит передать свое состояние другой анон
421 458153
>>58151

> Обязательно должно быть либо фермион-бозон, либо фермион-бозон-фермион.



W, Z, и глюоны смотрят на тебя
422 458154
>>58151
Не совсем полный ответ у тебя, как я уже сказал выше, гамма-гамма рассеяние возможно - через создание электрон-позитронной пары (или вообще лептонной/кварковой пары, неважно), просто вероятность крайне мала. Хотя на LHC, по-моему, всё же наблюдали. Конечно, для видимого света энергий не хватит для создания пары.
423 458156
Вам не кажется это неправильным, что фотоны вот так нагло пересекаются с друг другом и от этого нет никакого эффекта
424 458157
>>58156
Не нравится - едь в другую Вселенную, где фотоны реагируют.
425 458162
Возникла тут такая мысль. Известно, что ячейки флэш-памяти со временем деградируют пропорционально количеству раз перезаписи.
Я так понимаю, это связано с изменением свойств диэлектрика, который пробивается каждый раз для записи.

Интересно, а что, если не использовать диэлектрик вообще, а вместо него использовать вакуум, а чтобы помещать заряды на проводник, находящийся в вакууме, использовать какой-нибудь эффект вроде термоэлектронной эмиссии?
426 458163
>>58156
фотоны не упругие шарики. они вообще мало общего с шариками имеют.
427 458164
>>58151

>Частицы не могут непосредственно сами с собой


ну и фотон-фотон непосредственно не взаимодейсвтуют друг с другом, если под непосредственно имеется в виду 3х хвостка, чего не бывает.
428 458173
Может ли человек в эхо локацию? Позволяют ли ушные раковины это дело? Сколько лет минимум нужно для развития этого навыка?
429 458177
>>58173
У слепых людей что-то такое получается, читал где-то.
430 458178
>>58177
в спид-инфо?
431 458181
На основании чего учёные утверждают что вселенная расширяется путём растягивания пространства, а не того что все объекты просто разлетаются?
Потому что нет центра расширения? Но если вселенная очень большая, то будет похоже что объекты разлетаются одновременно все ото всех, если рассматривать относительно маленькую область для того чтобы сделать такой вывод.
По краснодарскому смещению?
А что-то кроме есть? Что-то ещё
432 458182
>>58181

>краснодарскому


Лол блжад
игорь антихайп 433 458184
сап. 10 класс. Хочу вкатиться в математику. С чего начать. Посоветуйте учебники/сайты для изучения.
434 458186
>>58184

>сап. 10 класс. Хочу вкатиться в математику.


Да всё равно ЕГЭ профильное по математике не сдашь, иди на менеджера по туризму учись.
435 458189
>>58164
четырехвостка через фермионный квадрат же
436 458190
>>58186
По себе скажу, что в 10 классе читал М.Гарднера, Я.Перельмана и какой-то учебник по матану для ВТУЗов. Не знаю есть ли чего лучше сейчас.
Программировать учись еще - в любом случае пригодится.
437 458201
Возможно ли при помощи генной инженерии вывести новый вид человека с придатками вырабатывающими электричество? По типу тех,что у скатов
438 458203
>>58201
Можно, но выйдет говно без задач, поскольку проебем работу других органов.
Да и выработка градиента потенциала на клетках довольно энергетически затратная штука, для млекопитающих это непозволительная роскошь, чтоб тратить на бесполезную хуиту.
439 458204
>>58203
Этот придаток может реанимировать сердце, при недостатке кислорода
440 458205
>>58204
Этот придаток первый откажет при недостатке кислорода.
441 458211
>>58204

>Этот придаток может реанимировать сердце, при недостатке кислорода


Ебло, чтобы реанимировать сердце, нужно сперва устранить причину его остановки.
442 458212
>>58211

>если бы придаток был сформирован изначально ни у кого бы не возникло вопросов


Ты ж мань не знаешь про выброс гормонов , в критическом состоянии организма, подмойся мань
443 458217
>>58212
Если у тебя нет кислорода, выброс гормонов никак не поможет.
444 458221
Какие открытия последних 3-5 лет наиболее важные/прорывные?
445 458222
>>58221
Это сказать нельзя сразу. Обычно когда что-то стреляет в практическом плане лет через 10, все такие "ууу какое прорывное открытие мы сделали".
446 458223
>>58221
Спрашивай отрасль. Сейчас наука настолько специализирована, что сложно себе представить какое то открытие, которое внезапно стало бы важным и прорывным для всех остальных направлений. Как Бозон Хиггса. Ни для кого кроме физиков элементарных частиц это открытие не является прям важным.
447 458237
>>58223
Бозон Хиггса не был прорывным открытием, заебали. Прорывным было (для теоретичекской физики) открытие хиггсовского механизма спонтанного нарушения калибровочной симметрии. Бозон Хиггса был лишь одним из следствий теории (позже ставшей частью Стандартной модели). Его экспериментальное открытие ничего не изменило. Если бы бозонов оказалось много, то тогда да, СМ пошла бы по пизде и это был бы охуенный прорыв. Но бозон оказался один, причем в точности такой, как его предсказывала теория. Но так как СМ и так уже было подтверждена с охуительной точностью в других экспериментах, на хиггса вскоре стало всем похуй.
448 458242
>>58221
Те, что за рамками СМ.
1. Открытие массы у нейтрино.
2. Открытие (вроде как) стерильных нейтрино.

Техническое открытие, которое позволяет видеть нейтрино без их поглощения:
Упругое когерентное рассеяние нейтрино
449 458243
>>58237

>Если бы бозонов оказалось много


А может ли оказаться несколько вариантов, только этого одного бозона?
450 458245
>>58243
смотря что за них считать, так-то и у электрона куча вариантов – с разным зарядом и разной хиральностью, т.е. всего 4 частицы под одним названием, если позитрон (антиэлектрон) не считать.
Но H - это античастица сама себе, а без спина у него и хиральности нет, т.е. он один, насколько я понимаю.

давно замечаю, что чем спрашивать у местных физиков, проще написать хероту, чтобы они потом поправляли с горящими глазами
451 458249
>>57703

>Ибо тут нужен качественный революционный скачок в аккумуляторах. А его не предвидется.


А почему? Есть что почитать на эту тему?
452 458250
есть несколько вопросов к спецам по deep learning от обычного хомячка обывателя.
На данный момент сеть может справляться достойно с одной двумя задачами, вроде распознавания голоса или классификации объектов по изображению с камеры,но есть проблемы в том, что тот же AI на машинах не понимает связи между детьми на обочине и мячом улетевшим на дорогу.
Почему не сделать древовидную структуру из множества сетей?
Нейросети низшего звена занимаются обработкой первичных сигналов : звук, свет. Сети рангом выше занимаются классификацией объектов, атрибутами окружающей среды. В итоге на конечную сетку передаются не графические данные, а смысловые конструкции.
fallout-76-vault-boy-thumbs-up.jpg.optimal.jpg69 Кб, 1080x600
453 458252
>>58249
Потому что запас энергии в аккуме не может быть больше, чем запас энергии в бензине, т.к. там всё – химические связи.
Только карманные атомные реакторы решат вопрос.
454 458253
>>57703

>А его не предвидется.


он произошел еще до 2010-го года.
но вам его не покажут ближайшие лет 300, пока уголь не перепродадут с нефтью в тридорога.
455 458257
>>58221
Открытие гравитационных волн.
456 458258
>>58253
Чертовы жидомассоны, все в генератор каланадзе.
457 458259
>>58250
Начнем с твоего посыла.

> не понимает связи между детьми на обочине и мячом


Что значит "не понимает"? Если в тренировочном датасете
есть множество ситуаций с детьми и мячами, то сетка вполне
даже выучит чего ты там от нее хочешь "понимать".

То что сетка не обладает человеческой полнотой знаний
об окружающем мире. Так тебя отправь на планету Двачанию
и там также будут говорить "он не понимает связи между
грумяблями на вычючине и куйцом - неразумный, хули с него взять".
458 458262
>>58258
глупо верить, что в мире, где под предлогом "государственной безопасности" убивают безнаказанно, элита вдруг озаботится проблемами нищих.
бюджеты, выделяемые на оборонку всеми странами исчисляются триллионами. вы что, всерьез думаете, что они все полученные данные публикуют в открытой печати?
особенно имеющие стратегическую важность?
а юморок ваш объясняется тем, что вы не знаете что такое допуски, какие они бывают и что бывает с теми, кто не следуют правилам.
потому что сами вы как бы никто и звать вас никак и загрифованных документов для таких как вы не существует.
459 458266
>>58252

>Потому что запас энергии в аккуме не может быть больше, чем запас энергии в бензине, т.к. там всё – химические связи.


Охуеть, в бензине массой со стандартную ячейку 18650 энергии в 48 раз больше чем в лучших образцах.
460 458267
>>58259
Проблема как раз в том, что ситуации с детьми есть. И машина остановится. Но у нее нет навыков предугадывать поведение личинок.
Опять пример с личинкой, уже без мяча - падает игрушка на проезжую часть, личинус рвется туда прямо под машину - опасность есть.
Либо взрослый кидает пустую пачку сигарет на дорогу - опасности нет. С точки зрения машины разницы в этих двух событиях 0
461 458269
>>58267
Наверное душу их не чувствует, потому что сама машина бездуховная, не то что духовный водитель.
462 458270
>>58267

>Но у нее нет навыков предугадывать поведение


В тренировочных данных были примеры различных "поведений", которые надо было предугадывать?
15142894176680.jpg326 Кб, 800x530
463 458271
>>58262
Тайсон в лекции или в книге говорил мол ученые не могут скрывать что-то. Это сама суть ученых. Только они открывают что-то новое, то им сразу надо написать статью и напечататься в крутом научном журнале.
464 458274
>>58271
В последнее время все совсем не так. Есть шанс крупно обосраться, как уже было со сверхсветовыми нейтрино, аномалией Пионера, 750GeV-бозоном и поляризацией реликтового излучения. Поэтому ходит немало слухов про якобы-открытия, которые были сделаны, но так и не вышли далеко за пределы исследовательских коллабораций, т.к. заветные 5σ так и не удалось добрать.

У теоретиков с этим проще, т.к. экспериментальных подтверждений в ближайшем будущем нет и не предвидится. Но там тоже дохуя результатов гуляет неопубликованными - их кто-нибудь доложил на семинаре с 3,5 участниками, но по тем или иным причинам не потрудился записать в виде, который можно было хотя бы запостить в архив.
15409681275380.jpg108 Кб, 799x534
465 458275
>>58274
Ну значит открытия хуевые, которые никому не интересны. Низкосортные.
466 458286
>>57355
>>57406
>>57589
Как и ожидалось, кроме пустых кукареков никаких информативных возражений небыло.
Сознаниепетухи такие сознаниепетухи, пздц.
467 458288
>>58286
А что еще ответить маньке которая не может хотя бы пару статей базовых прочесть?
Биологический нейрон и отходящие от него дендриты гораздо более глубокие, их нельзя представить в виде 0 и 1.
Начни с биохимии. Нейрон не может быть включен или выключен.
468 458291
>>58288

>Нейрон не может быть включен или выключен.


Сигнал от него может идти или не идти, или идти с разной силой, какие-то ещё варинты есть?
Подумаешь не по одному входу-выходу.

Да, 0 и 1 это испускание и не испускание сигнала, кстати, а не включеность или выключеность логического элемента, если ты не знал.
469 458293
>>58291

> какие-то ещё варинты есть?


Охохохо... там дохуя еще вариантов, начиная от гормонального состава, заканчивая разными сигналами с помощью разных нейромедиаторов. (Ты их количество видел?)
470 458296
>>58291

>Сигнал от него может идти или не идти


Питурд может срать или не срать, питурда можно обозначить 1 битом.
471 458297
>>58293

>Охохохо... там дохуя еще вариантов, начиная от гормонального состава, заканчивая разными сигналами с помощью разных нейромедиаторов. (Ты их количество видел?)


>Йа сматрйю картенки на комплюктере и не вижу там никоких нулей и еденичек, я вижу картенки, вы ебанутые?? Кокие нули и еденицы


Ты рассматриваешь более высокие уровни.
472 458298
анон хватит агрессировать и иди учи базовые основы биохимии мозга, тогда тебе все станет ясно.
473 458299
>>58298

>биохимии мозга


Зачем учить частный случай физики?
474 458306
Если люди не вылупляются из яиц, то почему близнецов называют однояйцевыми?
475 458308
>>58296

>питурда можно обозначить 1 битом


Слишком дохуя информации, в случае питардиев-ананизмусов типа вас всех хватит и сотых долей бита.
476 458317
>>58275

>Ну значит открытия хуевые, которые никому не интересны. Низкосортные.



ОК. Раз так сказал щкольник со щмищными картиночками на сцаче, так тому и быть!
477 458319
>>58317
Мамку ебал.
15010981713870.jpg62 Кб, 753x648
478 458320
>>58317
Ну, какой вывод еще можно сделать, если открытия сделаны, но никому нахуй не нужны, даже самим открывателям? Если они так сильно бояться обосраться, что нихуя не делают со своим открытиями?
479 458321
>>58320

>какой вывод еще можно сделать


Вывод что тебе пора делать уроки
480 458332
>>58237
Я и написал "важным".
481 458334
>>58297
Нет. Я просто не застрял в 19-ом веке, когда машина Тюьринга типа может симулировать сознание.
482 458335
>>58291
Ой, бля. Вчера мозг был компьютером, сегодня уже нейронной сетью. Уже давно обосрали эту, иди на ютуб.
948329986w640h6403prostoppryladmm7.jpg35 Кб, 450x450
483 458340
1. Заливаем в пикрил чистую воду.
2. Включаем.
3. Ставим эту йобу в морозильник на минус дохуя градусов
Я думаю, что вода не замёрзнет ни при каком охлаждении. Я прав?
484 458341
>>58340

>вода не замёрзнет ни при каком охлаждении


На основании чего?
485 458347
>>58334

>симулировать сознание.


А что это такое?
Или проблема сознания сложная и ты не можешь ответить на этот вопрос?
486 458349
>>58335

>Ой, бля.


Какой ты нервный и неустойчивый, это потому что твоя верадостаточно пустышка?

Если ты не заметил, вопрос был про нули и единицы, и как вообще оно может функционировать, если не на них.
487 458351
>>58349
Нет там нулей и едениц. Там есть бессчётное количество значений, которые хоть и можно вписать в значения от нуля до единицы, но это всё равно далеко от двоичной системы, ибо значений может быть миллионы и плюс есть другие факторы, из-за которых ответ на тот же самый сигнал при одинаковых значениях нейрона может быть различным.
488 458352
>>58349
Ну и полный ноль нейрон сможет принять если только он умрёт.
489 458353
>>58351

>Там есть бессчётное количество значений


Есть ли разница между этими значениями? Или без разницы какое значение идет на вход?
490 458355
>>58353
Это значение может говорить о чувствительности нейрона к сигналу.
Что касается сигнала, то ещё надо не забывать, что он проходит двумя абсолютно разными способами - химическим и электрическим.

Если уж охота мозг с чем-либо сравнивать, то ближе всего к его работе интернет, а не нейронная сеть и тем более компьютер.
491 458356
>>58353
Если речь про потенциал действия, то без разницы какая амплитуда, она всегда примерно одинаковая для одной и той же клетки. Кодирование сообщений происходит за счет частоты.
Но есть и другие, graded potentials, которые могут быть различной величины и это имеет значение, потому что сигналы складываются.
492 458357
>>58353
>>58355
Все равно 0 и 1.
Все просто, есть потенциал действия - это передача 1. Потенциал покоя это 0, Критический потенциал это смена 0 на 1. До критического потенциала будет 0, после физически может быть только 1.
493 458358
>>58351

>Там есть бессчётное количество значений, которые хоть и можно вписать в значения от нуля до единицы, но это всё равно далеко от двоичной системы, ибо значений может быть миллионы


Аналоговое клонение к да или нет? Ок.

>и плюс есть другие факторы, из-за которых ответ на тот же самый сигнал при одинаковых значениях нейрона может быть различным.


Что за они? Ты про те что просто влияют на склонение к да или нет?
494 458359
>>58357

>Все равно 0 и 1.


Не, он походу про одновременныую подачу сигнала "разной силы" по разным ветвям, а с тех ветвей по тем ветвям, а с тех по тем... в итоге чего, в конечном ответе даже череда довольно большого числа значений "ближе к нет чем остальные" может в итоге привести к "да", в отличии от других, в которых было больше "да"

Хотя хз нельзя ли это считать двоичной логикой.
Что довольно легко ею это можно описать так точно(но не воссоздать, из-за огромного числа ветвлений), но вот можно ли считать это двоичной..
495 458360
>>58357
Логика блондинки же. Типа победе в лотерее 50/50, ведь либо да, либо нет.
496 458361
>>58355

>Если уж охота мозг с чем-либо сравнивать, то ближе всего к его работе интернет, а не нейронная сеть и тем более компьютер.



Вот, кстати, да - у большинства людей именно как интернет мозг работает. Точнее даже как рунет - с блокировками, роскомнадзором, отставанием в развитии от остальной сети на пару лет, срачами, тараканами, зародышевым мировоззрением и т.д.
Но если ты имеешь в виду именно модель процессов, то можешь конечно попытаться натянуть модель оси на свой головной мозг (или что там у тебя вместо биологической нейронной сети - набор протоколов?), но я все таки обоснованно считаю, что моя биологическая нейронная сеть работает именно как нейронная сеть, блядь.
497 458363
>>58361

>моя биологическая нейронная сеть работает именно как нейронная сеть


Нет. Нейронка слишком примитивная. Не в размере даже, а тупо по количеству функций самих нейронов, и способов взаимодействий с другими.
498 458364
>>58358

>Что за они? Ты про те что просто влияют на склонение к да или нет?


Нейромедиаторы. Есть просто, что отвечают за торможение и возбуждение нейрона, в добавок есть ещё те, которые влияют на более тонкую настройку работа мозга.
>>58361
Нейросеть это в первую очередь обычный алгоритм, где весь сигнал идёт по прямой. Он вдохновлялся работой нейронов, но общего с мозгом у него столько же сколько у самолёта с птицей, а то и меньше.
499 458365
>>58363

>Нейронка слишком примитивная. Не в размере даже, а тупо по количеству функций самих нейронов, и способов взаимодействий с другими.


Фигасе ты расфантазировался
Верун-нью дженерэйшон.
500 458366
>>58364

>Нейромедиаторы. Есть просто, что отвечают за торможение и возбуждение нейрона, в добавок есть ещё те, которые влияют на более тонкую настройку работа мозга.


Можно было и не упоминать.
501 458367
>>58361
Свободный мозг от блокировки - это например мозг наркомана при ломке. При ней активность нейронов, что отвечают за чувствительность к сигналам боли стремится к 1 и пропускает абсолютно все сигналы, от чего болеть начинает каждая клетка тела. Торможение и блокировка в мозгу более важный процесс, ибо по дефолту значение всех нейронов это 1, ведь 0 только у мёртвого нейрона. Но пропуск всех возможных сигналов и постоянное активное возьуждение нейронов это путь к скорой смерти
502 458368
>>58363
Нет. Протез руки тоже примитивный по сравнению с биологической рукой. Но это не говорит о том, что биологическая рука работает скорее как двигатель внутреннего сгорания, а не как механический протез.

Вообще, у тебя какая-то дичь в твоем головном интернете. Искусственные нейронные сети появились как результат изучения нервной активности. И это не "мозг работает как нейронная сеть" - мозг и есть нейронная сеть, а искусственная нейронная сеть - это то, что мы можем создать на основе изучения мозга и вообще всей нс.

>>58364

>Нейросеть это в первую очередь обычный алгоритм, где весь сигнал идёт по прямой


Серьезно? Понятно.
503 458370
>>58367

>Свободный мозг от блокировки - это например мозг наркомана при ломке.


Это такой лютый бред, что я пожалуй соглашусь, чтоб ты нахуй заткнулся.
504 458371
>>58359
Ну я понял, что имеется в виду. Дело то в том, что сигнал либо есть, либо его нет. Система направлена не на суммирование мелких сигналов. Обычно есть доминирующий сигнал, который вставочными нейронами супрессирует остальные сигналы, это можно реализовать логическими вентелями. А если есть случай для включения нейрона от нескольких акснов, то это просто логическая операция AND, для простоты понимания в рамках двоичной логики, происходит не 0.33+0.33+0.33 = 1, а будет что, 1 AND 1 AND 1. Сами нейроны работают не по принципу аккумулирования и удержания сигнала, а по принципу частоты дискретных сигналов. А вот за рабочую частоту клеток отвечает их лабильность.
Все таки тут чистая двоичная логика.
505 458372
>>58370
лол
506 458374
>>58371

> это можно реализовать логическими вентелями


А чем реализуешь эти вентили?
507 458375
>>58371
Хм, я думал типа идёт по одной цепочке 0.1, 0.1, 0.1, а потом хуяк и 0.7
А по второй стабильно больше шло в среднем 0.2, 0.2, 0.2, 0.2
В итоге в первой 1, а во второй всего 0.8 и как ответ выбрана первая, хоть и лидировала вторая почти до конца вторая.
508 458376
>>58374

>А чем реализуешь эти вентили?


Ору.
Паскаль в школе ещё не начинали читать?
509 458377
>>58376
Тебе придётся делать вентили для вентилей же и так дальше добавлять ещё сотни тысяч костылей.
510 458378
>>58371

>логическая операция AND


Точно AND а не OR? Каким образом реализуется И?
511 458382
>>58377

>Тебе придётся делать вентили для вентилей же и так дальше добавлять ещё сотни тысяч костылей.


И? В этом и сложность мозга, в количестве, а не сложности.
Тупо количестве.
512 458383
>>58375
Ну потенциалу достаточно появиться в одном месте для лавинообразного распространения. А сам сигнал не может быть 0.3 или 0.8 или 1.2. Когда достгается критический потенциал, тогда запускается потенциал дейтсвия и это 1. Если не достигается, то клетка не возбуждается и это 0. Сами само значение потенциала не несет значения, -70 милливольи или -69 милливольт важно лишь само действие. Логика возбудимых тканей приспособлена на передачу дискретных потенциалов.

>>58378
В клетках AND будет является накопительным, скажем интенсивность высвобождения нейромедиаторов или активирования большего числа потенциалозависимых помп. По своей сути это все равно будет являть конъюнкцией.
И да, скорее OR будет, если рассматривать дискретные импульсы от аксонов. Я просто показавал, что все это можно применить в рамках двоичной логики. Но чтобы не быть голословным клетку можно активировать мембранным потенциалом соседних клеток, что можно рассматривать как AND или через суммирования локальных ответов, что может привести к деполяризации клетки.

>>58375
В этом кстати и заключается OR.
Как я написла выше

>Все просто, есть потенциал действия - это передача 1. Потенциал покоя это 0, Критический потенциал это смена 0 на 1. До критического потенциала будет 0, после физически может быть только 1.


просто 0.8 в твоем понимании будет до критического потенциала, т.е 0
1 это критичекий потенциал, т.е 1.
Если они оба выше критического потенциала, то который первее, так как после уже не важна сила, важна интенсивность. Но клетка не сможет возбуждаться быстрее лабильности, так же при долгом стимулировании клетки возникает адаптация, что уменьшает ее лабильность.
513 458384
>>58382
Ну ты хотел сделать подобие же мозга, а не попытку его проэмулировать. Тебе для того, что бы описать мозг в двоичной системе не хватит ни коддеров, которые трудились бы над этим за еду круглые сутки в течении десятков лет, ни мощностей всех компьютеров разом.

Твоя нейросеть должна копировать работу мозга, а не пытаться её эмулировать.
514 458387
>>58384

>Ну ты хотел сделать подобие же мозга, а не попытку его проэмулировать.


Ты спросил как реализовать вентили, как будто это неебичиски тонкие материи, я тебе ответил что это легко делается на языке программирования который изучают школьники.
515 458390
>>58387

>легко


лол.

Не работает наш мозг ни на каком программном языке и не сможешь ты эти вентили им описать. Разумеется имеется в виду описать корректно. Есть ты утверждаешь, что мозг это нейросеть, то ты и должен его воссоздавать лишь на базе одной нейросети, а не добавляя убогий паскаль и прочие программные языки.
516 458391
>>58390

>лол


Серьёзно, поучи паскаль, полезное дело.
517 458392
>>58391
Паскаль умер же уже.
518 458393
>>58391
поизучай лучше ты, в добавок изучи нормально нейронные сети на пару с работой мозга. Там с одним гипоталамусом столько проблем, что каким только изобретением его не пытались называть.
519 458395
Как можно с биологической точки зрения описать тягу к своему же полу?
520 458397
>>58393
Приведи пример вентиля, который нельзя будет реализовать в паскале.

>>58392

>Паскаль умер же уже.


Умер, но это не мешает ему продолжать работать.
image.png297 Кб, 756x522
521 458398
>>58391
Ты скатываешь всё в спор не о мозге, а в спор о симуляции физических систем на машине Тьюринга. А это уже около-философский вопрос На самом деле нет, привет теоремы Гёделя и неисчислимые задачи. Ты просто какой-то рафинированный позитивист. Такие разве еще существуют?
522 458400
>>58374
Всмысле чем, в железе транзисторами. В програмных языках логическими операциями.
523 458404
>>58398 (Я не тот анон если что, просто мимо проходил)
Сначала подумал что это тот дед, что топит за свехтьюринга (забыл как там его), либо хуеплет Пенроуз. И действительно - хуеплет Пенроуз. Столько буков, кручу верчу наебать хочу. Ну так что в итоге там может или не может, можешь прямо ткнуть в ответ? А если и может то хули с того, но это уже задачка со звездочкой.

>привет теоремы Гёделя


Мммм... главное правило теоремы Геделя - когда нечего спиздануть - спиздани про теорему Геделя.
524 458405
>>58404

>Мммм... главное правило теоремы Геделя - когда нечего спиздануть - спиздани про теорему Геделя.


Усвоено!
мимодругойанон
525 458413
>>58398

>Ты просто какой-то рафинированный позитивист


Ликбезните меня по этой вашей фейлософии, если позитивисты

>определяют единственным источником истинного, действительного знания эмпирические исследования


То что тогда другие, достают их из собственной сраки?
526 458414
>>58397
на паскале хоть матрицу можно реализовать. при условии, что мощность твоего компьютера пожирает ресурсы большей части вселенной.
527 458418
>>56196 (OP)
Поясните за атомы и пустоту между ними/между ядром и нейтронами.
Я тут дрочил и слушал Карла Сагана, а он вдруг пизданул, что материя в большинстве своем состоит из нихуя. Почему тогда есть агрегатные состояния вещества? Если пустота внутри атома значительно больше непустоты, то как оно все не развалилось нахрен?
528 458422
>>58341
На основании того, что мне это трудно представить ) Ну неужели это не верно? Разве могут крупные кристаллы образовываться в сильно турбулентных течениях?
529 458423
>>58418
квантовый мир далеко не такой какой обычный. поэтому и эта "пустота" далеко не такая, как тебе кажется.
530 458424
>>58418
Есть материя, а есть вещество. "Нихуя" - это поле, оно отличается от вещества тем, что всюду плотно и непрерывно и пронизывает вещественные тела. Общие черты - наличие энергии. (Но это классические представления.)
.jpg541 Кб, 1920x1080
531 458429
>>58418
Дело в том что эта пустота не совсем пустота. Как бы тебе понятнее рассказать... вот в небе бывают облака и временами из них даже дождик капает, а бывает нихуя - чистое небо, вроде ничего там и нету на первый взгляд, пустота одна. Вот в таком же смысле пустота межу атомов - пустая на первый взгляд, но не пустая по сути. Наши предки ее еще называли словом таким прикольным, забористым, как же его... блин, забыл.
15484388848280.png61 Кб, 736x736
532 458434
>>58429
Ну так то пустоты вообще не бывает. Масса, как минимум, созает пространство.
533 458440
>>58422

>Разве могут крупные кристаллы образовываться в сильно турбулентных течениях?


Что в твоём понимании крупный кристалл? Полностью замёрзшая эта баклажка?
534 458445
Ребят, есть вопрос.
Мы можем откачать из сосуда весь воздух и получить вакуум для проведения экспериментов.
Но возможно ли создание небольшого пространства с нулевой гравитацией? Там где Земля, и остальные объекты не будут никак влиять.
535 458447
>>56196 (OP)

Почему нельзя свет представить продольной волной в некотором пространстве вместо изобретения костылей в виде фотонов с нулевой массой, световой скоростью и остановившемся для него временем? Вы тут вообще в курсе про две скорости звука для газов?
Стикер191 Кб, 500x500
536 458452
>>58445
Cамолет в свободном падении.
537 458455
>>58447

>Почему нельзя свет представить продольной волной в некотором пространстве вместо изобретения костылей в виде фотонов с нулевой массой,


Можно. Просто чтобы подключить квантовую механику, тебе приходится это поле квантовать.
538 458459
>>58447
Потому что продольная волна уже подразумевает участки неоднородной плотности в чем-то. Тебе для этого приедтся придумывать костыли типо эфира. Так же как-то описать его поляризацию. Все попытки описания света волнами либо корпускулами по отдельности не увенчались успехом, и не увенчаются тем боле сейчас в эпоху квантовой теории поля. Мы совершили концептуальный скачек, обратно никак, можно лишь совершить еще один вперед. И этот скачек очень важен, так как показывает, что наше мышление не привязано к аналогиям наблюдаемыми в нашем мире, а способен предсказать что-то абстрактное, что будет являться верным.
И фотон, это не костыль, это наблюдаемый объект.
539 458485
Если опустить железку в сверхкислоту, она быстрее растворится? Намного? Или эта сверхкислотность имеет смысл лишь в контексте кислотного пвп кто кому протон всучит (или третьему агенту)?
540 458487
>>58459

>Так же как-то описать его поляризацию.



Не вижу проблемы, вот есть нечто, что является атомом, на самом деле это градиент быстро переходящий в более плотную область, у него есть момент вращения, когда волна бьёт по нему от другого объекта, он или сжимается, а потом расширяется на величину равную длине волны , а момент его вращения накладывается на него, или тоже самое но с тем отличием, что световая волна это сферическая спираль с шагом, равным опять же длине волны уходящая от него.

И да, мне понятно, что ща будут истории про то, что я ничего не понимаю и наука только для посвящённых, но я так и не услышал ответа на свой вопрос. Что я имею ввиду? Хотя бы немного физических явлений, что с вашей точки зрения не могут быть описаны некой средой, в моём случае газом. Про поляризацию я понял, правда проблемы я в ней не вижу. Есть же что-то ещё. Напиши плес. И да, я не тралль, мне действительно важно наконец узнать пусть и целый список того, что не может быть описано светоносной средой.
541 458489
>>58487
Момент вращения, если я правильно понял, то получится поперечная волна же, т.е. классическая физика.
Поэтому я не понимаю что не ясно?
Так и есть, эл.м излучение - поле, а его квант - фотон. Фотон физически наблюдаем и хорошо описывается квантовой теорией. Просто непонимание может быть связано с "корпускулярно-волновой дуализм" где, якобы, говорится, что это и то и другое, хотя на деле это ни то и ни другое.
542 458490
>>58489

>Момент вращения, если я правильно понял, то получится поперечная волна же, т.е. классическая физика.



А разве в 3х мерном пространстве такая волне не будет распространяться как сфера? Я это к чему, среда - газ, с определённой, большой плотностью, но очень малой частицей, как итог нету анизотропии и волна-возмущение-смещение распространяется как расширяющаяся сфера с определённой толщиной, ну и убывает как положено обратно пропорционально квадрату расстояния. Разве не так волны распространяются в газовых средах?
543 458491
>>58490
Ну тебе придется брать 4-х мерное, так как СТО.
изображение.png463 Кб, 1312x720
544 458492
>>58491

Вот что я имею ввиду, ядро условно, ядро того же водорода вместе с электроном.

Касаемо четвёртого измерения то ты же прекрасно понимаешь, что я его не учитываю. Насколько помню оно называется Евклидовым, вот в нём я и рассматриваю данный процесс. Что я хочу от тебя услышать. Почему это неверно? В каких случаях это не сработает?
545 458497
>>58440
Просто ответь, замёрзнет или нет
546 458500
>>58490

>А разве в 3х мерном пространстве такая волне не будет распространяться как сфера?


Электромагнитное излучение может распространяться как сфера, но не обязано.
Лазер или антенны тому подтверждение. Все равно она остается поперечной, а не продольной.
547 458501
>>58492
с ядром совсем все по другому
548 458510
>>58445

>возможно ли создание небольшого пространства с нулевой гравитацией?


Вот ты сам же и ответил на свой вопрос:

>Там где Земля, и остальные объекты не будут никак влиять.


То есть в межзвездном пространстве, на максимальном удалении от любых массивных объектов.

Но тут надо учитывать, что если

>откачать из сосуда весь воздух и получить вакуум


Это будет нихуя не "нихуя". И даже если взять космический вакуум в жопе холодильника эридана - все эти квантовые флуктации, бозоны хиггса с ненулевым вакуумом, и гравитационные волны от масштабных событий будут нехило заполнять это "нихуя", хоть обосрись.
Кстати, если бы кефирщики не пустили по пизде свою теорию, и занимались ее развитием, а не пиздежом и срачами, они бы приплыли к модели хиггса намного раньше, ящитаю.

А тебе зачем, вообще? Какие эксперименты над "гравитационным вакуумом" ты собрался ставить, если еще нет ни одного эксперимента, подтверждающего вообще существование гравитона. Только ОТО с парой альтернативных теорий, и вирго с лиго, которые детектят волну, но ничего не могут сказать о частице-переносчике.
549 458533
>>58500

>Все равно она остается поперечной, а не продольной.



А не может ли быть причиной этому большая скорость?
550 458590
>>58533
Нет, большая скорость лишь следствие безмассовости переносчиков энергии электромагнитного поля.
Я понял, что не совсем коректно понимаешь поперечность и продольность волны.
Для продольной волны очень важна среда, так как распростронение ее фронта связано с отклонением, скажем, узлов от состояния эквилибриума по направлению распространения волны.
Электромагнитная поперечная волна представляется как такая синусойда колеблющаяся вверх вниз, как веревка, но это не так. У такой волны есть направленность, но нет масштабных параметров, кроме "длины волны". Она колеблется в пространстве Электрического потенциала и Магнитной индукции перпендикулярно, а я не Y и Z. Тут сложно представить все это дело если не знаешь, что ткаое векторное поле. Она в этом смысле поперечная.
Тут нет колебания узлов стреды от точки эквелибриума, тут есть, грубо говоря векторное поле, и в каждой его точке по направлению распространения волны, меняются энергия поля.
551 458607
>>58590
И вот поэтому я и стал задавать эти вопросы. А именно, зачем такие сложности? Неужели описание ЭМ волны именно как классической продольной волны? С моей точки зрения это ооооочень сильно упростит жизнь и наконец откроется то, как можно в космосе передвигаться. Я читал, что ультра звуковые волны ведут себя подобно частицам, намёк совершенно ясный.

Какое явление природы показывает невозможность такого объяснения световой волны?
552 458643
>>58607
Потому что это не так. Подобные терии давно были придуманы и описаны, но они не сопдадают с природой. Тогда не было бы смысла плодить разные волны, если их можно было бы точно описать одной.
Продольная волна будет означать наличие среды, состоящей из частиц, имеющая узлы способные колебаться. Продольная волна имеет пространсвтенные параметры в соответствии с размерностью пространства и поэтому имеет, им определяемые, стемени свободы. И все равно придется вводить кучу разных параметров, способ их описания, физику на том как они дейсвтуют, и самое главное, переписать все последующие теории, при наличии того, что есть эксперементальные данные доказывающие их правдивость.
ЭМ волны это лишь удобное описание в классической физики, и это не есть природа, так же как Ньютоновская механика.
553 458814
>>56594
У Фарадея спроси
554 458831
>>58497
Замерзнет
555 458842
Возможно ли в теории взаимодействия и манипуляция с самим пространством?
556 458869
>>58842
Как только поймём, как пользоваться гравитонами, так сразу
557 459527
Помогите решить зодачу, пожалуйста.
Внутри откачанной до глубокого вакуума установки находится герметичный теплонепроницаемый цилиндрический сосуд, заполненный идеальным одноатомным газом. Сосуд закрыт сверху теплонепроницаемым поршнем, на котором стоит гиря. Объём, занимаемый газом, равен при этом V. Гирю с поршня снимают. Найдите объём газа в новом положении равновесия. Массы поршня и гири одинаковы.
Я пытался решить так: т. к. имеем теплонепроницаемые цилиндр и поршень, то процесс будет адиабатным. Для него верно: p V5/3= const. Так как гирю сняли, то оказываемое давление уменьшилось в два раза, значит, величина V5/3 увеличилась в два раза. В итоге V1 = V 81/5. А в задачнике написано, что V1 = 8V/5. ЧЯДНТ?
558 459575
>>59527

>то процесс будет адиабатным


Это было бы, если бы поршень двигался очень медленно. А если гирю сняли - поршень стал двигаться с ускорением и набирал существенную скорость в процессе перехода к новому состоянию равновесия - всё, процесс не квазистатический и адиабатой не является.
559 459600
>>59527
Если что, решил тряхнуть и нашкрябал тут в пейнте некоторую шнягу. Могу выложить если очень нужно.
560 459660
>>59600
Выложи, пожалуйста

>>59575
Если не считать процесс адиабатный, то я не знаю, как её решить. Она школьная же.
561 459666
>>59660

>Если не считать процесс адиабатный, то я не знаю, как её решить.


Процесс не адиабатный, зато общая энергия системы (газ+поршень) у газа внутренняя, у поршня потенциальная сохраняется в начале, когда сняли гирю - и в конце, когда получилось новое равновесие + условие, что давление упало в джва раза.

Кстати, сейчас подумол, что если бы масса газа была сравнима с массой поршня, пришлось бы учитывать изменение потенциальной энергии газа (в смысле, в поле силы тяжести), но массу газа без его молярной массы не посчитать - а она не дана, значит подразумевается, что надо пренебречь.
562 459742
Можно было бы это поэлегантнее оформить как по внутренней так и по внешней форме, но уж не обессудь
563 463477
всем hi. на связи студент-биотехнолог, отчаянно ищущий помощи. пишу диплом по агротрансформации табака, скоро словлю нервный срыв, так как немного (много) всего недопонимаю, во всей теме не вижу общей картины, сроки поджимают, а науч.рук изысканно игнорит мои вопросы уже месяц. буду рад как щенок, которого подобрали добрые руки с улицы в теплый дом, если на мою просьбу откликнется знающий человек, не против потратить лишнюю минутку на такого додю как я...
Тред утонул или удален.
Это копия, сохраненная 5 мая 2019 года.

Скачать тред: только с превью, с превью и прикрепленными файлами.
Второй вариант может долго скачиваться. Файлы будут только в живых или недавно утонувших тредах. Подробнее

Если вам полезен архив М.Двача, пожертвуйте на оплату сервера.
« /sci/В начало тредаВеб-версияНастройки
/a//b//mu//s//vg/Все доски